Advanced Physical Assessment Exam 1

Ace your homework & exams now with Quizwiz!

32 yo office worker reports excessive stress at work, pain in RLQ, states last night she vomited 2x, BP 120/75, HR 93, looks pale & is sweating lightly, which is objective finding?

Accelerated heart rate Being pale isn't objective since it may not be pale to another clinician

"Instability in interpersonal relations, self-image, and affective regulation, impulsivity." describes which personality disorder?

Borderline personality

What sounds are suggestive of asthma?

Wheezes Also in COPD or bronchitis Mediastinal crunch: pneumomediastinum Pleural rub: pleural effusion, pneumothorax Ronchi: secretions in larger airways Stridor: partial obstruction of larynx or trachea

Very sensitive methods for detecting hearing loss include which of the following?

Audiometric testing While it is important to screen for hearing complaints with methods available to you, it should be realized that some physical examination techniques are limited. Nonetheless, you should be comfortable performing these tests, as audiometric testing is not always available.

Which of the following statements is true concerning mental health disorders in primary care?

Mood disorders make up ~25% of all diagnoses

Mr. Curtiss has a history of obesity, diabetes, osteoarthritis of the knees, HTN, and obstructive sleep apnea. His BMI is 43 and he has been discouraged by his difficulty in losing weight. He is also discouraged that his goal weight is 158 pounds away. What would you tell him?

"Even a weight loss of 10% can make a noticeable improvement in the problems you mention."

14yo male presents to new PCP after family relocates to new state, underwent tx for sarcoma when he was 11yo, including above-the-knee amputation, has learned to successfully navigate with prosthetic leg & even engage in competitive athletics at school, he does not like to speak of his experience with cancer and often makes up humorous stories to tell new acquaintances about his amputation ("I got bit my squirrel and they had to amputate it"), although he is very well engaged for most of the visit with the new clinician, when the topic of cancer arises, he demures to his father, who accompanies him to this appointment. Which of the following statements is most likely to be helpful in cementing the pts trust in the new provider?

"That sounds like a frightening experience that you have recovered well from."

You have just asked a patient how he feels about his emphysema. He becomes silent, folds his arms across his chest and leans back in his chair, and then replies, "It is what it is." How should you respond?

"You seem bothered by this question."

A 30-year-old sales clerk comes to your office wanting to lose weight; her BMI is 30.0kg/m2 What is the most appropriate amount for a weekly weight reduction goal?

.5-1lb per week

39M 2 hx fever, chills, cough, productive green sputum, dyspnea, hx of serious illness, T 101.2, other VSS, late inspiratory crackles in LLL, EE sounds like "AA", percussion would be:

dull s/s PNA, consolidation, dullness over area of PNA Flatness over muscles Tympany: gastric air bubble

Weight change may indicate the presence of important underlying pathology requiring further investigation, which of the following best describes significant weight change that requires further evaluation?

A 45M with baseline weight of 280lbs who decides to undertake a light exercise regimen and loses 15% of his total body weight in 3 months A safe rate of intentional weight loss is no more than 2lb/wk; this pt has lost about 3.5lb/wk

70yo man complains of double vision, which of the following associated s/s would be worrying about an underlying neurological problem (as opposed to pathology) in the eye?

Abnormality in EOMs on examination paralysis or weakness of EOM suggests possible brainstem or cerebellar lesion

72yo retiree presents to cardiology clinic with palpitations after several months of s/s. An EKG shows tachyarrythmia, which cardiologist dx as a-fib. In measuring BP of pt with chronic a-fib, which of the following is true?

Ambulatory monitoring over 2-24 hours is recommended DT rhythm producing variable & inconsistent BPs

Which of the following anatomic landmark associations is correct? A) 2nd intercostal space for needle insertion in tension pneumothorax B) T6 for lower margin of endotracheal tube C) Sternal angle marks the 4th rib D) 5th intercostal space for chest tube insertion

A Chapter: 08 Page and Header: 283, Anatomy and Physiology Feedback: The 2nd intercostal space is indeed the correct location for insertion of a needle in tension pneumothorax. The other answers are incorrect. T4 marks the approximate bifurcation of the trachea and therefore marks the inferior limit for an endotracheal tube on chest X-ray. The sternal angle marks the 2nd rib, which helps establish the 2nd interspace for needle insertion as above or locations for cardiac auscultation (aortic and pulmonary areas). Finally, the 4th intercostal space is normally used for chest tube insertion.

448. All of the following are associated with emphysema except:

A CXR with infiltrates and flattening of costovertebral angle Emphysema is characterized by having a barrel-shaped chest, pursed-lip breathing, and dyspnea when at rest. Infiltrates on an x-ray indicate bacterial infection, such as pneumonia.

For which of the following patients would a comprehensive health history be appropriate?

A new patient with the chief complaint of "I am here to establish care" Patient is new

249. All of the following clinical findings are considered benign oral findings except:

A patch of leukoplakia Leukoplakia mainly affects the mucous membranes of the mouth. It is thought to be caused by irritation. Leukoplakia are patcheson the tongue, in the mouth, or on the inside of the cheek that occur in response to long-term irritation, including smoking, holding chewing tobacco or snuff in the mouth for a long period, or other tobacco use, especially pipes (smoker's keratosis). Leukoplakia on the tongue is also an early sign of HIV.

162. When assessing a patient suspected of having vertigo, which description provided by the patient is most consistent with the diagnosis?

A sensation of spinning or rotating Vertigo is defined as having a sensation of spinning or rotating.

111. A 28-year-old male nurse of Hispanic descent reports a history of a cold that resolved 2 weeks ago except for a dry cough and pain over his right cheek that worsens when he bends down. The patient denies fever. He tells the employee health nurse practitioner that he is very allergic to both cephalexin (Keflex) and erythromycin. The patient's vital signs are temperature of 99.2°F, pulse of 72 beats/min, and respirations of 12 breaths/min. Which of the following conditions is most likely?

Acute sinusitis Acute sinusitis symptoms include cough, facial pain, and low- grade fever.

230. A 12-year-old girl is complaining of a 2-week history of facial pressure that worsens when she bends over. She complains of tooth pain in her upper molars on the right side of her face. On physical exam, her lung and heart sounds are normal. Which of the following is the most likely diagnosis?

Acute sinusitis Signs and symptoms of acute sinusitis include headache, facial pain that worsens with bending over, eye/ear pressure and pain, aching in upper jaw/teeth, reduced smell and taste, cough (especially at night due to the nasal drainage), sore throat, bad breath, and fatigue.

104. All of the following are known to cause chronic cough except:

Acute viral URI Chronic cough can be caused by chronic bronchitis, allergic rhinitis, and gastroesophageal reflux disease (GERD). The cough associated with a viral URI is a mild to moderate hacking cough. The cough is usually dry (no sputum). With postnasal drip, the cough may bring up some nasal secretions. The cough abates with the viral illness.

159. The mother of a 4-week-old infant is concerned that her infant's eyes are crossed for a few seconds occasionally. The nurse practitioner would:

Advise the mother that this is a normal finding in infants up to 2 months of age

151. A nurse practitioner is doing a funduscopic exam on a 35-year-old woman during a routine physical exam. He notices that she has sharp disc margins and a yellowish- orange color in the macular area. The ratio of veins to arteries is 3:2. What is the next most appropriate action?

Advise the pt that she had a normal exam The optic disc of a normal examination has sharp margins, a yellowish-orange to a creamy pink color, and round or oval shape. To test, have the patient look at a distant fixed point and direct the light of the ophthalmoscope at the fundus. The ophthalmoscope (set at +8 to +10) should be close to your eyes; your head and scope move together. Check for the red reflex, then adjust the diopter setting; approach more closely to inspect the optic disc, veins, arteries, and the macula. The veins are darker in color and larger than the arterioles (3:2 ratio).

55. Which of the following actions is appropriate follow-up for this 70-year-old patient?

Advise the pt that the condition will resolve spontaneously Subconjunctival hemorrhages do not require any treatment. The blood in the eye will be absorbed within 10 to 14 days.

Which of the following is a "Red flag" presenting with headache?

Age over 50 A unilateral headache is often seen with migraines and may commonly be accompanied by phonophobia and photophobia. Pain over the sinuses from sinus congestion WWW.TESTBANKTANK.COM may also be unilateral and produce pain. Migraine and sinus headaches are common and generally benign. A new severe headache in someone over 50 can be associated with more serious etiologies for headache. Other red flags include: acute onset, "the worst headache of my life"; very high blood pressure; rash or signs of infection; known presence of cancer, HIV, or pregnancy; vomiting; recent head trauma; and persistent neurologic problems.

The CAGE questionnaire is a short screening examination administered in the office to evaluate for which of the following?

Alcohol misuse

You are performing a young woman's first pelvic examination. You make sure to tell her verbally what is coming next and what to expect. Then you carry out each maneuver of the examination. You let her know at the outset that if she needs a break or wants to stop, this is possible. You ask several times during the examination, "How are you doing, Brittney?" What are you accomplishing with these techniques?

All: Increasing the patient's sense of control Increasing the patient's trust in you as a caregiver Decreasing her sense of vulnerability

13yo girl BIB mother before start of 8th grade, 3d hx episodes of SOB, tingling around lips, no fever, cough, sputum production, CP, no hx of serious illness, no meds, VSS, cardiac lung and extremity WNL. dx?

Anxiety

Abstract thinking is an important is an important component of the human thought process. A person's ability to understand questions that tests his or her ability to answer appropriately is dependent upon a number of factors. Which one of the following answers is true in identifying a pt with concrete thinking and a reduced ability to think abstractly?

An inability to discern the similarity between two words (a cat and a mouse by answering "the cat chases the mouse"). An inability to link a cat and a mouse as both representing animals (abstraction) is an example of loss of ability to think abstractly.

Diplopia, which is present with one eye covered, can be caused by which of the following problems?

An irregularity in the cornea or lens Double vision in one eye alone points to a problem in "processing" the light rays of an incoming image. The other causes of diplopia result in a misalignment of the two eyes.

A 68-year-old retired postman presents to your clinic, complaining of dull, intermittent left-sided chest pain over the last few weeks. The pain occurs after he mows his lawn or chops wood. He says that the pain radiates to the left side of his jaw but nowhere else. He has felt light-headed and nauseated with the pain but has had no other symptoms. He states when he sits down for several minutes the pain goes away. Ibuprofen, Tylenol, and antacids have not improved his symptoms. He reports no recent weight gain, weight loss, fever, or night sweats. He has a past medical history of high blood pressure and arthritis. He quit smoking 10 years ago after smoking one pack a day for 40 years. He denies any recent alcohol use and reports no drug use. He is married and has two healthy children. His mother died of breast cancer and his father died of a stroke. His younger brother has had bypass surgery. On examination you find him healthy-appearing and breathing comfortably. His blood pressure is 140/90 and he has a pulse of 80. His head, eyes, ears, nose, and throat examinations are unremarkable. His lungs have normal breath sounds and there are no abnormalities with percussion and palpation of the chest. His heart has a normal S1 and S2 and no S3 or S4. Further workup is pending. Which disorder of the chest best describes these symptoms?

Angina Pectoris Angina causes dull chest pain felt in the retrosternal area or anterior chest. It often radiates to the shoulders, arms, neck, and jaw. It is associated with shortness of breath, nausea, and sweating. The pain is generally relieved by rest or medication after several minutes. This patient needs to be admitted to the hospital for further workup for his accelerating symptoms.

A 55-year-old smoker complains of chest pain and gestures with a closed fist over her sternum to describe it. Which of the following diagnoses should you consider because of her gesture?

Angina pectoris The clenched fist of Levine's sign, while not completely specific for ischemic pain, should definitely cause you to consider this etiology. Bronchitis is usually painless and pericarditis can produce a sharp pain which worsens with inspiration. This is called pleuritic pain and can be associated with pneumonia and other chest diseases. Costochondritis is a parasternal pain, usually well localized. It is exquisitely tender.

87. This case is followed by questions and multiple-choice answers specific to the case. A 55-year-old male patient describes an episode of chest tightness in his substernal area that radiated to his back while he was jogging. It was relieved immediately when he stopped. The patient's symptoms are highly suggestive of what condition?

Angina pectoris The classic pain of angina is described as discomfort, pressure, tightness, or heaviness on the center or left side of the chest that is precipitated by exertion and relieved by rest. The pain can be referred to the back, shoulders, neck, or jaw. The pain is not sharp, knife-like, or stabbing in nature. Angina is caused by transient myocardial ischemia. The most common cause of angina is coronary artery disease. If the angina worsens, is not relieved by rest, or lasts more than 20 minutes, it may be due to acute myocardial infarction (MI). If acute MI is suspected, call 911.

312. This case is followed by questions and multiple-choice answers specific to the case. The nurse practitioner notices a gray ring on the edge of both irises of an 80-year-old woman. The patient denies visual changes or pain. She reports that she has had the "ring" for many years. Which of the following causes is most likely?

Arcus senilis Arcus senilis (arcus senilis corneae) is a white, gray, or blue opaque ring in the corneal margin (peripheral corneal opacity), or white ring in front of the periphery of the iris. It is present at birth, but then fades; however, it is commonly present in older adults. It can also appear earlier in life as a result of hypercholesterolemia. It does not affect vision. Unilateral arcus is a sign of decreased blood flow to the unaffected eye due to carotid artery disease or ocular hypotony.

A 23-year-old graduate student comes to your clinic for evaluation of a urethral discharge. As the provider, you need to get a sexual history. Which one of the following questions is inappropriate for eliciting the information?

Are you sexually active?

When you enter your patient's examination room, his wife is waiting there with him. Which of the following is most appropriate?

Ask if it's okay to carry out the visit with both people in the room

On a very busy day in the office, Mrs. Donelan, who is 81 years old, comes for her usual visit for her blood pressure. She is on a low-dose diuretic chronically and denies any side effects. Her blood pressure is 118/78 today, which is well-controlled. As you are writing her script, she mentions that it is hard not having her husband Bill around anymore. What would you do next?

Ask why Bill is not there

A 55-year-old bookkeeper comes to your office for a routine visit. You note that on a previous visit for treatment of contact dermatitis, her blood pressure was elevated. She does not have prior elevated readings and her family history is negative for hypertension. You measure her blood pressure in your office today. Which of the following factors can result in a false high reading?

Blood pressure cuff is tightly fitted

Adam is a very successful 15-year-old student and athlete. His mother brings him in today because he no longer studies, works out, or sees his friends. This has gone on for a month and a half. When you speak with him alone in the room, he states it "would be better if he were not here." What would you do next?

Assess his suicide risk His lack of interest in usual activities and duration of symptoms should make you suspicious for depression. Despite his very successful academic and athletic performance, you should recognize this last phrase indicating suicide risk. You could ask if he has had thoughts about hurting himself and, if so, how he would carry this out. Ask about firearms and other weapons at home. Adam needs immediate psychiatric referral if these risks are found, or admission to the hospital for observation if referral is not available in a timely fashion.

A 21-year-old college senior presents to your clinic, complaining of shortness of breath and a nonproductive nocturnal cough. She states she used to feel this way only with extreme exercise, but lately she has felt this way continuously. She denies any other upper respiratory symptoms, chest pain, gastrointestinal symptoms, or urinary tract symptoms. Her past medical history is significant only for seasonal allergies, for which she takes a nasal steroid spray but is otherwise on no other medications. She has had no surgeries. Her mother has allergies and eczema and her father has high blood pressure. She is an only child. She denies smoking and illegal drug use but drinks three to four alcoholic beverages per weekend. She is a junior in finance at a local university and she has recently started a job as a bartender in town. On examination she is in no acute distress and her temperature is 98.6. Her blood pressure is 120/80, her pulse is 80, and her respirations are 20. Her head, eyes, ears, nose, and throat examinations are essentially normal. Inspection of her anterior and posterior chest shows no abnormalities. On auscultation of her chest, there is decreased air movement and a high-pitched whistling on expiration in all lobes. Percussion reveals resonant lungs. Which disorder of the thorax or lung does this best describe?

Asthma Asthma causes shortness of breath and a nocturnal cough. It is often associated with a history of allergies and can be made worse by exercise or irritants such as smoke in a bar. On auscultation there can be normal to decreased air movement. Wheezing is heard on expiration and sometimes inspiration. The duration of wheezing in expiration usually correlates with severity of illness, so it is important to document this length (e.g., wheezes heard halfway through exhalation). Realize that in severe asthma, wheezes may not be heard because of the lack of air movement. Paradoxically, these patients may have more wheezes after treatment, which actually indicates an improvement in condition. Peak flow measurements help to discern this.

Pt has SOB, dullness to percussion over right base, breath sounds absent at RLL, no crackles, no wheezes or ronchi, no transmitted voice sounds, dx?

Atelectasis Also seen in pleural effusion L-HF: would have late inspiratory crackles PNA: also have late inspiratory crackles, with transmitted voice sounds Pneumothorax: would have hyperresonance COPD: hyperresonance

Mrs. Lenzo weighs herself every day with a very accurate balance-type scale. She has noticed that over the past 2 days she has gained 4 pounds. How would you best explain this?

Attribute this to body fluid

16. A 28-year-old student is seen in the school health clinic with complaints of a hacking nonproductive cough, rhinorrhea, pharyngitis, and malaise for the past 2 weeks. He does not take any medications, denies any allergies, and has no significant medical history. Physical examination reveals a low-grade temperature of 99.9°F, respirations of 16 breaths/min, pulse of 90 beats/min, and scattered rales and wheezing of the lungs. The patient does not appear toxic. The total white blood cell count is 10,500/μL. What is the most likely diagnosis?

Atypical pneumonia The most common organism causing community- acquired atypical pneumonia is Mycoplasma pneumoniae, an atypical bacterium. The populations with the highest infection rates are college students, school-aged children, and military recruits. Respiratory symptoms (cough) are accompanied by pharyngitis, rhinorrhea, and sometimes ear pain. It is easily spread from aerosol droplets.

143. All of the following pulmonary tests require the patient's voice to perform correctly except:

Auscultation a technique used to evaluate lung sounds, whereas tactile maneuvers use the hands. Fremitus occurs as a result of vibration of the chest wall when a person speaks and is heard through a stethoscope. Tactile fremitus is a type of vocal fremitus found over the area of secretions. Tactile fremitus is evaluated using the surface of both hands over the back/lungs. In bronchophony (the ability to hear increased loudness of the spoken sounds), even a whisper can be heard through the stethoscope.

Mrs. H. comes to your clinic, wanting antibiotics for a sinus infection. When you enter the room, she appears to be very angry. She has a raised tone of voice and states that she has been waiting for the past hour and has to get back to work. She states that she is unimpressed by the reception staff, the nurse, and the clinic in general and wants to know why the office wouldn't call in an antibiotic for her. Which of the following techniques is not useful in helping to calm this patient?

Avoiding admission that you had a part in provoking her anger because you were late

488. An older man is diagnosed with conductive hearing loss in the left ear by the nurse practitioner. Which of the following is the expected result when performing a Rinne test on this patient?

BC > AC A normal result in the Rinne test is air conduction (AC) greater than bone conduction (BC). When there is a conductive hearing loss (i.e., ceruminosis, otitis media), the result will be BC greater than AC. The reason is that the sound waves are blocked (i.e., cerumen, fluid in middle ear). Therefore, the patient cannot hear them as well as through bone conduction.

676. This case is followed by questions and multiple-choice answers specific to the case. A 17-year-old high school student is diagnosed by the nurse practitioner with serous otitis media of the left ear. He states that his nose is always congested because of dust mite allergy. His medical history includes allergic rhinitis and asthma. What is the expected result of the Rinne test in patients who have serous otitis media of the left ear?

BC > AC The sound is heard louder by bone conduction versus air conduction (BC>AC) in serous otitis media and other types of conductive hearing loss. This is because when the affected ear is filled with fluid, the tympanic membrane cannot move in response to the sound waves due to the fluid trapped inside the middle ear. Because the sound waves are blocked, the sound is heard better by bone conduction

245. A chest radiograph shows an area of consolidation on the lower lobe. Which of the following conditions is most likely?

Bacterial pneumonia Pneumonia is an inflammatory condition of the lung especially affecting the alveoli. It is associated with fever, chest symptoms, and consolidation on a chest x-ray. Infectious agents include bacteria, viruses, fungi, and parasites. Consolidation is not present in the lungs with bronchitis, chronic obstructive pulmonary disease (COPD), or atypical pneumonia.

324. A middle-aged hypertensive man presents to a public health clinic with complaints of an acute onset of fever, chills, and cough that is productive of rusty-colored sputum. The patient reports episodes of sharp pains on the left side of his back and chest whenever he is coughing. His temperature is 102.2°F, pulse is 100 beats/min, and blood pressure is 130/80 mmHg. The urinalysis does not show leukocytes, nitrites, or blood. These findings are most consistent with:

Bacterial pneumonia The most common cause of bacterial pneumonia is Streptococcus pneumoniae. Haemophilus influenzae, Chlamydia pneumoniae, Mycoplasma pneumoniae, and Legionella pneumophila are other important bacteria that cause pneumonia. Typically, pneumonia comes on very quickly; the patient has high fever/chills, productive cough with yellow or brown sputum, shortness of breath, and may have chest pain with breathing/coughing. The gold standard for diagnosing bacterial pneumonia is the chest x-ray, which shows infiltrates and/or lobar consolidation Older people can have confusion or a change in their mental abilities. In patients with bacterial pneumonia, it is important to determine whether bacteria are present in the urine, in order to identify appropriate antibiotics to treat the bacteria.

A 49-year-old administrative assistant comes to your office for evaluation of dizziness. You elicit the information that the dizziness is a spinning sensation of sudden onset, worse with head position changes. The episodes last a few seconds and then go away, and they are accompanied by intense nausea. She has vomited one time. She denies tinnitus. You perform a physical examination of the head and neck and note that the patient's hearing is intact to Weber and Rinne and that there is nystagmus. Her gait is normal. Based on this description, what is the most likely diagnosis?

Benign positional vertigo This is a classic description of benign positional vertigo. The vertigo is episodic, lasting a few seconds to minutes, instead of continuous as in vestibular neuronitis. Also, there is no tinnitus or sensorineural hearing loss as occurs in Ménière's disease and acoustic neuroma. You may choose to learn about Hallpike maneuvers, which are also helpful in the evaluation of vertigo.

Disparities in pain tx have been well described in numerous studies comparing Caucasian pts to those of African American & Hispanic origin. Which of the following statements is true concerning this issue?

Biases of the treating clinician are associated with under-treatment of pain in minority patients and non-English speakers

You are beginning the examination of a patient. All of the following areas are important to observe as part of the General Survey except:

Blood pressure

19yo student of art history presents to clinic after syncopal episode at school, notably thin, on thorough review of medical hx, admits he eats only minimally to maintain very low body weight that he feels is ideal, embarrassed that his issues were discussed by his peers after this episode, especially bc he believes that this is a problem only faced by girls and women. Concerning the two most common eating disorders (anorexia nervosa and bulimia nervosa), which of the following statements is true?

Both of these eating disorders are associated with real or imagined fear of being fat

Mrs. T. comes for her regular visit to the clinic. She is on your schedule because her regular provider is on vacation and she wanted to be seen. You have heard about her many times from your colleague and are aware that she is a very talkative person. Which of the following is a helpful technique to improve the quality of the interview for both the provider and the patient?

Briefly summarize what you heard from the patient in the first 5 minutes and then try to have her focus on one aspect of what she told you.

A 72-year-old African-American male is brought to your clinic by his daughter for a follow-up visit after his recent hospitalization. He had been admitted to the local hospital for speech problems and weakness in his right arm and leg. On admission his MRI showed a small stroke. The patient was in rehab for 1 month following his initial presentation. He is now walking with a walker and has good use of his arm. His daughter complains, however, that everyone is still having trouble communicating with the patient. You ask the patient how he thinks he is doing. Although it is hard for you to make out his words you believe his answer is "well . . . fine . . . doing . . . okay." His prior medical history involved high blood pressure and coronary artery disease. He is a widower and retired handyman. He has three children who are healthy. He denies tobacco, alcohol, or drug use. He has no other current symptoms. On examination he is in no acute distress but does seem embarrassed when it takes him so long to answer. His blood pressure is 150/90 and his other vital signs are normal. Other than his weak right arm and leg his physical examination is unremarkable. What disorder of speech does he have?

Broca's aphasia

Which lung sound possesses the characteristics of being louder and higher in pitch, with a short silence between inspiration and expiration and with expiration being longer than inspiration? A) Bronchovesicular B) Vesicular C) Bronchial D) Tracheal

Bronchial These sounds are consistent with bronchial breath sounds. Be alert for these, as they may occur elsewhere and indicate a pneumonia or other pathology. The current explanation for this phenomenon is that the sound from the trachea is carried very well to the chest wall by fluid. This same explanation explains "ee" to "aa" changes, whispered pectoriloquy, bronchophony, and other circumstances in which high-frequency sounds, normally blocked by the air-filled alveoli, could be transmitted to the chest wall.

240. An infant who does not have a history of reactive airway disease and allergy has both inspiratory and expiratory wheezing accompanied by fever and profuse clear nasal discharge. Which of the following is most likely?

Bronchiolitis Symptoms of tracheobronchitis include prominent dry, nonproductive cough; later, coughing up phlegm is common. Bronchiolitis is a viral infection caused by respiratory syncytial virus (RSV), which is commonly seen during the winter/spring months in infants and young children. Typical signs/symptoms include fever and inspiratory/expiratory wheezing with clear drainage. Croup is a viral infection with a classic "barking" cough; the patient may have a runny nose, but typically no fever. When a child swallows a foreign object, choking, wheezing, and shortness of breath may occur, but no fever or clear drainage are present.

When crackles, wheezes, or rhonchi clear with a cough, which of the following is a likely etiology? A) Bronchitis B) Simple asthma C) Cystic fibrosis D) Heart failure

Bronchitis Adventitious sounds that clear with cough are usually consistent with bronchitis or atelectasis. The other conditions would not be associated with findings that cleared with a cough.

616. The Rinne and the Weber tests are used to assess which of the following cranial nerves (CNs)?

CN VIII The Rinne and Weber tests are used to assess cranial nerve VIII (acoustic nerve). The patient's hearing is tested by air conduction (Rinne and Weber) and bone conduction (Rinne only).

302. Which cranial nerve (CN) is being evaluated when Rinne testing is done?

CN VIII The acoustic nerve, cranial nerve VIII, is being evaluated when the Rinne test is performed. The Rinne test is performed by placing the base of a vibrating tuning fork against the patient's mastoid bone to evaluate bone conduction. Ask the patient to tell you when the sound is no longer heard. Then place the tuning fork in the front of the ear to evaluate air conduction. The air conduction should be twice as long as bone conduction.

A young woman undergoes cranial nerve testing. On touching the soft palate, her uvula deviates to the left. Which of the following is likely?

CN X lesion on the right The failure of the right side of the palate to rise denotes a problem with the right 10th cranial nerve. The uvula deviates toward the properly functioning side.

A 62-year-old construction worker presents to your clinic, complaining of almost a year of chronic cough and occasional shortness of breath. Although he has had worsening of symptoms occasionally with a cold, his symptoms have stayed about the same. The cough has occasional mucous drainage but never any blood. He denies any chest pain. He has had no weight gain, weight loss, fever, or night sweats. His past medical history is significant for high blood pressure and arthritis. He has smoked two packs a day for the past 45 years. He drinks occasionally but denies any illegal drug use. He is married and has two children. He denies any foreign travel. His father died of a heart attack and his mother died of Alzheimer's disease. On examination you see a man looking slightly older than his stated age. His blood pressure is 130/80 and his pulse is 88. He is breathing comfortably with respirations of 12. His head, eyes, ears, nose, and throat examinations are unremarkable. His cardiac examination is normal. On examination of his chest, the diameter seems enlarged. Breath sounds are decreased throughout all lobes. Rhonchi are heard over all lung fields. There is no area of dullness and no increased or decreased fremitus. What thorax or lung disorder is most likely causing his symptoms?

COPD This disorder is insidious in onset and generally affects the older population with a smoking history. The diameter of the chest is often enlarged like a barrel. Percussing the chest elicits hyperresonance, and during auscultation there are often distant breath sounds. Coarse breath sounds of rhonchi are also often heard. It is important to quantify this patient's exercise capacity because it may affect his employment and also allows you to follow for progression of his disease. You must offer smoking cessation as an option.

83. A 67-year-old woman with a 30-pack-year history of smoking presents for a routine annual physical examination. She complains of being easily short of breath and is frequently fatigued. Physical examination reveals diminished breath sounds, hyperresonance, and hypertrophied respiratory accessory muscles. Her complete blood count (CBC) results reveal that her hematocrit level is elevated. Her pulmonary function test (PFT) results show increased total lung capacity. What is the most likely diagnosis for this patient?

COPD a progressive lung disease that includes emphysema and chronic bronchitis. The most common risk factor for COPD is long-term cigarette smoking (80%-90%). Another cause is alpha-1 antitrypsin deficiency and chronic fume exposure. COPD is now the third leading cause of death in the United States. The three cardinal symptoms of COPD are dyspnea, chronic cough, and sputum production. The lungs are hyperinflated, which changes the shape of the chest and diaphragm, making the mechanics of breathing more difficult. Excess mucus and obstructed airflow from progressive thickening and stiffening of the airways diminish breath sounds. COPD creates a high hematocrit percentage due to chronic hypoxemia.

You are running late after your quarterly quality improvement meeting at the hospital and have just gotten paged from the nurses' station because a family member of one of your patients wants to talk with you about that patient's care. You have clinic this afternoon and are double-booked for the first appointment time; three other patients also have arrived and are sitting in the waiting room. Which of the following demeanors is a behavior consistent with skilled interviewing when you walk into the examination room to speak with your first clinic patient?

Calm

138. The red reflex examination is used to screen for:

Cataracts Instruct patient to look straight ahead and avoid moving the eyes. Use a direct ophthalmoscope (set at "0"), stand about 18 inches from the patient and shine the light directly on the eyes. Examine each eye, and then both eyes. A normal red reflex exam will show an orange to red color and round shape, with both eyes symmetrical. If the red reflex test of an infant shows white-colored reflection, rule out retinoblastoma or congenital cataract. Refer any patient with an abnormal light reflex to an ophthalmologist (American Academy of Pediatrics, 2008).

226. An asthmatic exacerbation is characterized by all of the following symptoms except:

Chronic coughing A patient experiencing respiratory distress from an asthmatic exacerbation presents with tachypnea (>20 breaths/min), tachycardia or bradycardia, cyanosis, and anxiety. The patient appears exhausted, fatigued, and diaphoretic and uses accessory muscles to help with breathing. Physical exam reveals cyanosis and "quiet" lungs with no wheezing or breath sounds audible. A "chronic" cough is not a symptom of an acute exacerbation but is commonly present in people with asthma Although cough usually accompanies dyspnea and wheezing, it may present in isolation as a precursor of typical asthmatic symptoms, or it may remain the predominant or sole symptom of asthma. Cough-variant asthma is a type of asthma in which the main symptom is a dry, nonproductive cough.

You are observing a patient with heart failure and notice that there are pauses in his breathing. On closer examination, you notice that after the pauses the patient takes progressively deeper breaths and then progressively shallower breaths, which are followed by another apneic spell. The patient is not in any distress. You make the diagnosis of:

Cheyne-Stokes respiration

The following information is recorded in the health history, "I feel really tired", what category?

Chief complaint

418. A cauliflower-like growth with foul-smelling discharge is seen during an otoscopic exam of the left ear of an 8-year-old boy with a history of chronic otitis media. The tympanic membrane and ossicles are not visible, and the patient seems to have difficulty hearing the nurse practitioner's instructions. Which of the following conditions is best described?

Cholesteatoma An abnormal skin growth in the middle ear behind the eardrum is called cholesteatoma. Repeated infections and/or a tear or pulling inward of the eardrum can allow skin into the middle ear. Cholesteatomas often develop as cysts or pouches that shed layers of old skin, which build up inside the middle ear. Over time, the cholesteatoma can increase in size and destroy the surrounding delicate bones of the middle ear, leading to hearing loss that surgery can often improve. Permanent hearing loss, dizziness, and facial muscle paralysis are rare, but can result from continued cholesteatoma growth.

42yo architect presents with widespread pain complaints, including HAs almost daily, pain at site of old MVA injury, generalized achiness and hypersensitivity throughout the body, he recounts first episodes of ongoing pain occurred in his early 20s, he has been to many practitioners over several years seeking a firm diagnosis and adequate tx of his complaints. Which of the following statements is true regarding chronic pain?

Chronic pain is defined as pain not due to cancer or recognized medical condition that persists for >3-6 mos

314. The cones in the retina of the eye are responsible for:

Color vision There are two types of photoreceptors in the human retina: rods and cones. Cones are active at higher light levels (photopic vision), and are capable of color vision and night vision, as well as high spatial acuity. The central fovea is populated exclusively by cones. There are three types of cones referred to as short- wavelength sensitive cones, middle-wavelength sensitive cones, and long-wavelength sensitive cones—S-cones, M-cones, and L-cones for short. Rods are responsible for vision at low light levels (scotopic vision). They do not mediate color vision, and have a low spatial acuity.

26M homeless presents for new patient evaluation at community health care center, hx of IVDA, contracted hepatitis C, suffers from uncontrolled asthma that he has had since childhood, tx including frequent doses of oral steroids when he cannot keep inhalers in his possession, 2 yrs ago was dx with bipolar disorder, on today's visit, main concern is abscess in right AC at heroin injection site, which is the following is best approach to health hx for this pt at first visit?

Comprehensive health history All new pts need this, HPI, PMH, family history, personal & social history & ROS

587. Which of the following structures of the eyes is responsible for color vision?

Cones Rods and cones are photoreceptor cells of the retina. The cones of the eyes are responsible for color vision. Cones are very sensitive to colors (red, blue, or green) and work better in brighter light. Rods are good for night vision and for vision in low-light conditions because they are sensitive to light and dark. To remember them, note that both cone and color start with the letters "co."

130. A 68-year-old woman with hypertension and diabetes is seen by the nurse 529 practitioner for a dry cough that worsens at night when she lies in bed. She has shortness of breath, which worsens when she exerts herself, and has gained 6 lbs during the past 2 months. Her pulse rate is 90 beats/min and regular. She is on a nitroglycerine patch and furosemide daily. The best explanation for her symptoms is:

Congestive Heart Failure In congestive heart failure (CHF), the heart's ventricular function is inadequate. Symptoms include fatigue, diminished exercise capacity, shortness of breath, hemoptysis, cough, orthopnea, hypertension, nocturnal dyspnea, and edema. The kidneys begin to lose their normal ability to excrete sodium and water, leading to fluid retention. Lung congestion/pulmonary edema causes shortness of breath and a decreased ability to tolerate exercise.

A light is pointed at a patient's pupil, which contracts. It is also noted that the other pupil contracts as well, though it is not exposed to bright light. Which of the following terms describes this latter phenomenon?

Consensual reaction The constriction of the contralateral pupil is called the consensual reaction. The response of the ipsilateral eye is the direct response. The dilation of the pupil when focusing on a close object is the near reaction. Accommodation is the changing of the shape of the lens to sharply focus on an object.

Common or concerning symptoms to inquire about in the General Survey and vital signs include all of the following except:

Cough

A patient complains of knee pain on your arrival in the room. What should your first sentence be after greeting the patient?

Could you please describe what happened?

709. Which of the following infections can cause a "barky" cough?

Croup The "barky" cough of croup is harsh, loud, and worse at night. Other symptoms are fever, hoarse or raspy voice, and stridor. A warm steamy room (let hot water run in shower with door closed) or a mist humidifier help. Croup is caused by several types of viruses, but parainfluenza types 1, 2, and 3 are the most common viruses.

A 54 yo diplomat working at the UN reports occassional CP and tightness particularly when stressed over work deadlines, he is 6f4i, he has T 98.6F, BP 140/78, cut over 1 eye that he says is from shaving, which is subjective information?

Cut over eye from shaving

273. Which of the following is considered an objective finding in patients who have a case of suppurative otitis media?

Decreased mobility of the TM as measured by tympanogram Acute suppurative otitis media is an acute infection affecting the mucosal lining of the middle ear and the mastoid air system. Suppurative stage: The tympanic membrane bulges and ruptures spontaneously through a small perforation in the pars tensa. Ear discharge is usually present. Diagnosis is usually made simply by looking at the eardrum through an otoscope. The eardrum will appear red and swollen, and may appear either abnormally drawn inward or bulging outward. Using the tympanogram with the otoscope allows a puff of air to be blown lightly into the ear. Normally, this should cause movement of the eardrum. In an infection, or when there is fluid behind the eardrum, this movement may be decreased or absent.

38yo accountant presents to office with series of generalized complaints, relates that he feels a loss of pleasure in daily activities, has difficulty sleeping, is experiencing problems making decisions, which of the following best explains the pts presentation?

Depression

29M electrician co persistent cough & wheezing, particularly during exercise, says he smokes "occasionally" but rarely that he needs to purchase cigarettes, mostly bums them, what is the best next step?

Determine # of packs per year

A 62-year-old smoker complains of "coughing up small amounts of blood," so you consider hemoptysis. Which of the following should you also consider?

Epistaxis When you suspect hemoptysis, you must consider other etiologies for bleeding. Commonly, epistaxis can mimic this as well as bleeding from the gastrointestinal tract. The other answers, although they involve bleeding, are contained or distant from the pharynx.

A 75-year-old retired teacher presents to your clinic, complaining of severe, unrelenting anterior chest pain radiating to her back. She describes it as if someone is "ripping out her heart." It began less than an hour ago. She states she is feeling very nauseated and may pass out. She denies any trauma or recent illnesses. She states she has never had pain like this before. Nothing seems to make the pain better or worse. Her medical history consists of difficult-to-control hypertension and coronary artery disease requiring two stents in the past. She is a widow. She denies any alcohol, tobacco, or illegal drug use. Her mother died of a stroke and her father died of a heart attack. She has one younger brother who has had bypass surgery. On examination you see an elderly female in a great deal of distress. She is lying on the table, curled up, holding her left and right arms against her chest and is restless, trying to find a comfortable position. Her blood pressure is 180/110 in the right arm and 130/60 in the left arm, and her pulse is 120. Her right carotid pulse is bounding but the left carotid pulse is weak. She is afebrile but her respirations are 24 times a minute. On auscultation her lungs are clear and her cardiac examination is unremarkable. You call EMS and have her taken to the hospital's ER for further evaluation. What disorder of the chest best describes her symptoms?

Dissecting aortic aneurysm A dissecting aortic aneurysm is associated with a ripping or tearing sensation that radiates to the neck, back, or abdomen. Because blood supply to the brain and extremities is disrupted, syncope and paraplegia or hemiplegia can occur. Blood pressure will usually be different between the two arms, and the carotid pulses often show an asymmetry. This is because the aneurysm decreases flow distally and causes inequality of flow between sides.

in the case of a middle-aged female with pounding headache, what is an effective question to ask the pt?

Does the pt have any aura prior to the headaches? Increase the likelihood that this is a migraine

514. A patient with chronic obstructive pulmonary disease (COPD) is referred for 592 pulmonary function testing. All of the following results are characteristic of pulmonary function tests in patients with COPD except:

Dyspnea Dyspnea is a symptom. The stem of the question is asking for pulmonary function test results, which includes TLC, RV, FEV1, and others. In chronic obstructive pulmonary disease (COPD), there is a reduction of the FEV1 (forced expiratory volume in 1 second) with increase in the TLC (total lung capacity) and RV (residual volume). The lungs of patients with emphysema have lost their elastic recoil (reduced FEV1). The lungs are always full of air that is hard to "squeeze out" of the lungs (this increases residual volume and total lung capacity). To summarize, COPD = reduction in FEV1 with increases RV and TLC.

A 35-year-old stockbroker comes to your office, complaining of feeling tired and irritable. She also says she feels like nothing ever goes her way and that nothing good ever happens. When you ask her how long she has felt this way she laughs and says, "Since when have I not?" She relates that she has felt pessimistic about life in general since she was in high school. She denies any problems with sleep, appetite, or concentration, and states she hasn't thought about killing herself. She reports no recent illnesses or injuries. She is single. She smokes one pack of cigarettes a day, drinks occasionally, and hasn't taken any illegal drugs since college. Her mother suffers from depression and her father has high blood pressure. On examination her vital signs and physical examination are unremarkable. What mental health disorder best describes her symptoms?

Dysthymic disorder

59M presents to PCP with several episodes of sharp epigastric pain, father died of pancreatic cancer at 52yrs, "his pain was just like mine is now..." pt pauses for several seconds, clinician replies, "just like?" after which pt restarts his narrative. Which of the following is an example of interviewing techniques employed by the clinician?

Echoing

A 29-year-old physical therapist presents for evaluation of an eyelid problem. On observation, the right eyeball appears to be protruding forward. Based on this description, what is the most likely diagnosis?

Exophthalmos Exophthalmos is the condition when the eyeball protrudes forward. If it is bilateral, it suggests the presence of Graves' disease. If it is unilateral, it could still be caused by Graves' disease. Alternatively, it could be caused by a tumor or inflammation in the orbit.

23F PA is nervous when she examines men her age, encountered young male who appeared embarrassed to see her when she walked in, what should she do to minimize mutual discomfort?

Explain how the exam will proceed

A patient presents with ear pain. She is an avid swimmer. The history includes pain and drainage from the left ear. On examination, she has pain when the ear is manipulated, including manipulation of the tragus. The canal is narrowed and erythematous, with some white debris in the canal. The rest of the examination is normal. What diagnosis would you assign this patient?

External otitis These are classic history and examination findings for a patient suffering from external otitis. Otitis media would not usually have pain with movement of the external ear, nor drainage unless the eardrum was perforated. In this case the examination of the eardrum is recorded as normal. Cholesteatoma is a growth behind the eardrum and would not account for these symptoms. Otitis media would classically be accompanied by a bulging, erythematous eardrum.

168. Which of the following would you recommend on an annual basis for an elderly patient with type 2 diabetes?

Eye exam with an ophthalmologist Elderly patients with type 2 diabetes should have a dilated eye exam done annually by an ophthalmologist. They should also see a podiatrist once or twice a year. Preventive care also includes receiving a flu shot annually, receiving a Pneumovax vaccine if older than 60 years of age, and taking a 81-mg baby aspirin each day.

Mr. W. is a 51-year-old auto mechanic who comes to the emergency room wanting to be checked out for the symptom of chest pain. As you listen to him describe his symptom in more detail, you say "Go on," and later, "Mm-hmmm." This is an example of which of the following skilled interviewing techniques?

Facilitation

42F mathematician persents for fu care regarding new dx of SLE 6 mos ago after lengthy dx process during which she was debilitated with fatigue & joint pain. Since dx, has been minimally compliant with meds & has switched her rheumatology provider twice, she continues to feel ill, in explanation for lack of adherence to prescribed tx, she simply says, "I don't like it". At this initial visit with her third rheumatology provider, clinician elects to explore the issues behind her noncompliance before engaging in diagnostics and tx using the FIFE model. Which of the following best defines the elements of the FIFE model?

Feelings, ideas, function and expectations Captures pts emotional landscape, intellectual landscape, current situation (function), and thoughts about future conditions.

A 29-year-old woman comes to your office. As you take the history, you notice that she is speaking very quickly, and jumping from topic to topic so rapidly that you have trouble following her. You are able to find some connections between ideas, but it is difficult. Which word describes this thought process?

Fight of ideas This represents flight of ideas because the ideas are connected in some logical way. Derailment, or loosening of associations, has more disconnection within clauses. Circumstantiality is characterized by the patient speaking "around" the subject and using excessive detail, though thoughts are meaningfully connected. Incoherence lacks meaningful connection and often has odd grammar or word use. Although severe flight of ideas can produce this condition, evidence is not present in this vignette.

How do you find the fourth intercostal space?

Find sternal angle & move finger laterally to 2nd rib, walks down to the 2nd intercostal space, third rib, third intercostal space, fourth rib & fourth intercostal space Sternal angle used as starting point for determining where second rib is and then one walks down from there to find intercostal spaces & other ribs

252. What is the best procedure for evaluating a corneal abrasion?

Fluorescein stain Fluorescein stain is an eye stain used to detect abrasions or foreign objects in the cornea of the eye. Orange dye (fluorescein) is used to stain the eye and a blue light is used to detect/visualize any foreign bodies or abrasions in the eye. Visual field test assesses vision. Tonometry measures the pressure inside the eye. The funduscopic exam is performed with an ophthalmoscope to visualize the inside of the eye.

34M hx complex social & medical needs (current substance abuse) presents to PCP teaching clnic, has experienced a # of adversarial relationships w prior clinicians, including voluntarily leaving 2 practices within the previous year & being asked to leave care at a third clinic due to misbehavior. The attending doctor desires to utilize the approaches to this pt that are most likely to lead to comprehensive care & pt compliance. Which of the following is most appropriate interview style to use?

Following the pts lead to understand their thoughts, ideas, concerns & requests

Instruct pt to take deep breath in, then with mouth open, breathe out as fast and completely as they can, what are you checking?

Forced expiratory time

593. Which structure of the eye is responsible for 20/20 vision (sharpest vision)?

Fovea of the macula The fovea is located in the center of the macula and is responsible for the sharpest vision ("20/20 vision") in the eyes. In the fovea, the only receptors are the cones, which allow us to see in color and in detail. The macula is responsible for central vision.

A 25-year-old radio announcer comes to the clinic for an annual examination. His BMI is 26.0 kg/m2 He is concerned about his weight. Based on this information, what is appropriate counsel for the patient during the visit?

Give the pt information about reduction of fat, cholesterol and calories because he is overweight

A grandmother brings her 13-year-old grandson to you for evaluation. She noticed last week when he took off his shirt that his breastbone seemed collapsed. He seems embarrassed and tells you that it has been that way for quite awhile. He states he has no symptoms from it and he just tries not to take off his shirt in front of anyone. He denies any shortness of breath, chest pain, or lightheadedness on exertion. His past medical history is unremarkable. He is in sixth grade and just moved in with his grandmother after his father was deployed to the Middle East. His mother died several years ago in a car accident. He states that he does not smoke and has never touched alcohol. On examination you see a teenage boy appearing his stated age. On visual examination of his chest you see that the lower portion of the sternum is depressed. Auscultation of the lungs and heart are unremarkable. What disorder of the thorax best describes your findings?

Funnel chest (pectus excavatum) Funnel chest is caused by a depression in the lower portion of the sternum. If severe enough there can be compression of the heart and great vessels, leading to murmurs on auscultation. This is usually only a cosmetic problem, but corrective surgeries can be performed if necessary.

137. A high school teacher complains of a dry cough for the past 6 weeks. It worsens when he is supine. He has episodes of nausea and heartburn, which he self-treats with an over-the-counter (OTC) antacid. He chews mints for his "bad breath." Which of the following is a possible cause for this patient's cough?

GERD Classic signs of gastroesophageal reflux disease (GERD) include acid reflux (regurgitation) into the esophagus, heartburn, and nausea. Complications include ulcers, esophageal strictures, Barrett's esophagus, cough, asthma, and throat or laryngeal inflammation. Risk factors include obesity, pregnancy, smoking, and alcohol use.

Suzanne, a 25 year old, comes to your clinic to establish care. You are the student preparing to go into the examination room to interview her. Which of the following is the most logical sequence for the patient-provider interview?

Greet the patient, establish rapport, invite the patient's story, establish the agenda, expand and clarify the patient's story, and negotiate a plan.

You note that a patient has anisocoria on examination. Pathologic causes of this include which of the following?

Horner's syndrome Anisocoria can be associated with serious pathology. Remember to exclude benign causes before embarking on an intensive workup. Testing the near reaction in this case may help you to find an Argyll Robertson or tonic (Adie's) pupil.

A 22-year-old man is brought to your office by his father to discuss his son's mental health disorder. The patient was diagnosed with schizophrenia 6 months ago and has been taking medication since. The father states that his son's dose isn't high enough and you need to raise it. He states that his son has been hearing things that don't exist. You ask the young man what is going on and he tells you that his father is just jealous because his sister talks only to him. His father turns to him and says, "Son, you know your sister died 2 years ago!" His son replies "Well, she still talks to me in my head all the time!" Which best describes this patient's abnormality of perception?

Hallucination

A 67-year-old retired janitor comes to the clinic with his wife. She brought him in because she is concerned about his weight loss. He has a history of smoking 3 packs of cigarettes a day for 30 years, for a total of 90 pack-years. He has noticed a daily cough for the past several years, which he states is productive of sputum. He came into the clinic approximately 1 year ago, and at that time his weight was 140 pounds. Today, his weight is 110 pounds. Which one of the following questions would be the most important to ask if you suspect that he has lung cancer?

Have you tried to lose weight?

59M unemployed co almost always feeling tired & hungry, despite getting sufficient rest & having good appetite & access to sufficient food, pt is obese, despite warm weather, wearing thermal socks with his sandals, says that is bc his feet are always cold & "feel funny", which body system do you begin assessing?

Head & neck Should proceed "head to toe" to optimize pt comfort, minimize # of changes in position, ensure nothing was missed.

A patient complains of shortness of breath for the past few days. On examination, you note late inspiratory crackles in the lower third of the chest that were not present a week ago. What is the most likely explanation for these? A) Asthma B) COPD C) Bronchiectasis D) Heart failure

Heart failure The timing of crackles within inspiration provides important clues. These late inspiratory crackles that appeared suddenly would be most consistent with heart failure. COPD and asthma usually produce early inspiratory crackles. Bronchiectasis, as seen in cystic fibrosis, classically produces mid-inspiratory crackles, but this is not always reliable. Interestingly, end-expiratory crackles can be heard in asthma on occasion.

Pt co epistaxis, which other causes should be concerned?

Hematemesis Although the source of epistaxis may seem obvious, other bleeding locations should be on the differential. Hematemesis can mimic this and cause delay in life-saving therapies if not considered. Intracranial hemorrhage and septal hematoma are instances of contained bleeding. Intestinal hemorrhage may cause hematemesis if there is obstruction distal to the bleeding, but this is unlikely.

You ask a patient to draw a clock. He fills in all the numbers on the right half of the circle. What do you suspect?

Hemianopsia You should suspect a visual problem because there is no writing on one half of the circle. This is consistent with a hemianopsia, sometimes seen in stroke. These patients may also eat food on only one half of their plate. The other conditions would not account for this pattern.

A young woman comes to you with a cut on her finger caused by the lid of a can she was opening. She is pacing about the room, crying loudly, and through her sobs she says, "My career as a pianist is finished!" Which personality type exhibits these features?

Histrionic The theatrical nature of her behavior as well as her overreaction lead to a diagnosis of histrionic character disorder.

A 12-year-old presents to the clinic with his father for evaluation of a painful lump in the left eye. It started this morning. He denies any trauma or injury. There is no visual disturbance. Upon physical examination, there is a red raised area at the margin of the eyelid that is tender to palpation; no tearing occurs with palpation of the lesion. Based on this description, what is the most likely diagnosis?

Hordeolum A stay, painful, tender, erythematous infection in gland at margin of the eyelid

192. A small abscess on a hair follicle on the eyelid is called:

Hordeolum Hordeolum is a common, painful, acute bacterial infection of the hair follicle on the eyelid. It is a focal infection (usually staphylococcal) involving either the glands of Zeis (external hordeola or styes) or, less frequent the meibomian glands (internal hordeola). Histologically, hordeola represent focal collections of polymorphonuclear leukocytes and necrotic debris (i.e., abscesses). Pinguecula is a thickening of the bulbar conjunctiva, located on the inner and outer margins of the cornea. Pterygium is a thickening of the conjunctiva located on the nasal or temporal cornea. Pinguecula and pterygium are both caused by the UV light of long-term sun exposure. Sunglasses with UV protection are recommended to prevent damage to the conjunctiva. Ptosis is the drooping of the upper eyelid.

394. A female patient complains of dizziness when she moves her head. You suspect benign paroxysmal positional vertigo. The diagnosis is supported by the presence of:

Horizontal nystagmus with rapid head movement Symptoms of benign paroxysmal positional vertigo include horizontal nystagmus with rapid head movement. Performing the Dix-Hallpike maneuver and/or the roll test will cause the symptoms of vertigo to appear.

A patient is examined with the ophthalmoscope and found to have red reflexes bilaterally. Which of the following have you essentially excluded from your differential?

Hypertensive retinopathy Hypertensive retinopathy requires a careful examination of the optic fundus. It cannot be diagnosed or excluded merely from the red reflex. Typically, the red reflex would be normal in this case. The other conditions are all associated with an abnormal red reflex.

A 27-year-old woman is brought to your office by her mother. The mother tells you that her daughter has been schizophrenic for the last 8 years and is starting to decompensate despite medication. The patient states that she has been taking her antipsychotic and she is doing just fine. Her mother retorts that her daughter has become quite paranoid. When asked why, the mother gives an example about the mailman. She says that her daughter goes and gets the mail every day and then microwaves the letters. The patient agrees that she does this but only because she sees the mailman flipping through the envelopes and she knows he's putting anthrax on the letters. Her mother turns to her and says, "He's only sorting the mail!" Which best describes the patient's abnormality of perception?

Illusion

a 24yo veteran returns from his second tour of duty in the Middle East. He was witness to a number of violent military encounters and experienced the death of several of his closest friends. He describes a number of problems including nightmares, poor sleep pattern, mild panic attacks, in persons with trauma & stress-related disorders as well as other disorders that may be associated with hallucinations and illusions, which of the following statements is true that distinguishes these two entities from each other?

Illusions are a misinterpretation of real stimuli, whereas hallucinations are subjective perceptions in the absence of real stimuli

PA had long day seeing a lt of pts, last pt is 80yo female BIB granddaughter, pts 1st visit, as part of hx PA obtains info about childhood & adult illnesses, moves onto family, which part did the PA leave out from past history?

Immunizations Social history is its own category

Glaucoma is the leading cause of blindness in African-Americans and the second leading cause of blindness overall. What features would be noted on funduscopic examination?

Increased cup-to-disc ratio It is important to screen for glaucoma on funduscopic examination. The cup and disc are among the easiest features to find. AV nicking and cotton wool spots are seen in hypertension. Microaneurysms are seen in diabetes.

543. All of the following physiological changes are present in the lungs of the elderly except:

Increased lung compliance Lung compliance decreases as we get older; therefore, the FEV1 also decreases. There is minimal change in the total lung volume. Airways tend to collapse earlier (than in young patients) with shallow breathing, which increases the risk of pneumonia.

715. An asthmatic 20-year-old woman who was seen for a viral upper respiratory 630 infection 2 weeks ago presents to the nurse practitioner's office complaining of a recent onset of shortness of breath, inspiratory and expiratory wheezing, and chest tightness. She has been using her albuterol inhaler four to six times a day with poor relief. She is unable to speak in full sentences. When the nurse practitioner quickly evaluates the patient, she notices that the patient is pale, diaphoretic, fatigued, and using her sternocleidomastoid accessory muscles for respiration. Her respiratory rate is 32 breaths/min and pulse is 130 beats/min. Which of the following interventions is not indicated?

Initiate CPR

511. Which of the following is involved with sensorineural hearing loss?

Inner ear Sensorineural hearing loss (i.e., presbycusis) involves damage to the hair cells in the cochlea (sensory portion) and may involve cranial nerve VIII (neural).

Correct order of performing lung examination?

Inspection, palpation, percussion, auscultation

Which of the following statements is true concerning the mini-mental status exam (MMSE)?

It is a proprietary screening test that is not diagnostic of probable causes

229. Atrophic macular degeneration of the aged (AMD) is the leading cause of blindness in the elderly in the United States. Which of the following statements is correct?

It is a slow or sudden painless loss of central vision Atrophic macular degeneration (AMD) occurs when the macula—the central portion of the retina that is important for reading and color vision—becomes damaged. This portion contains the fovea, with the largest number of cones in the eye, responsible for color vision and visual acuity. AMD is a single disease, but it can take two different forms: dry and wet. The atrophic or dry form of macular degeneration is the most common. There is a gradual withering of the visual cells and the blood vessels of the choroid (the vascular layer of tissue behind the retina). Usually the atrophic form results in only moderate loss of central vision. Although there is no medical or surgical treatment for the dry form of macular degeneration, eyesight may be helped somewhat by low-vision aids. These devices include magnifying lenses, brighter light for reading, or an electronic magnifier using a TV screen.

You are seeing an older patient who has not had medical care for many years. Her vital signs taken by your office staff are: T 37.2, HR 78, BP 118/92, and RR 14, and she denies pain. You notice that she has some hypertensive changes in her retinas and you find mild proteinuria on a urine test in your office. You expected the BP to be higher. She is not on any medications. What do you think is causing this BP reading, which doesn't correlate with the other findings?

It is caused by an "auscultatory gap."

630. While performing a funduscopic exam, the nurse practitioner notices arteriovenous (AV) nicking on the patient's retina. What causes AV nicking?

It is caused by an arteriole crossing a venule, which compresses the venule and causes it to bulge on each side AV nicking is one of the most common eye findings found in hypertensive retinopathy.

Concerning a patient that may demonstrate a diagnosis of aphasia, which of the following statements is true?

It is defined as an inability to produce or understand language two common types: receptive or expressive

Concerning hallucinations, an abnormal perception experienced by a patient, which of the following statements is true about this abnormality?

It may occur in association with a number of conditions including delirium and dementia, PTSD and schizophrenia

An 18-year-old college freshman presents to the clinic for evaluation of gastroenteritis. You measure the patient's temperature and it is 104 degrees Fahrenheit. What type of pulse would you expect to feel during his initial examination?

Large amplitude, forceful

Which of the following conditions would produce a hyperresonant percussion note? A) Large pneumothorax B) Lobar pneumonia C) Pleural effusion D) Empyema

Large pneumothorax There is a great deal of free air in the chest with a large pneumothorax, which produces a hyperresonant note. The other three conditions produce dullness by dampening the percussion note with fluid.

33. Human papillomavirus (HPV) infection of the larynx has been associated with:

Laryngeal cancer A) Laryngeal cancer Human papillomavirus (HPV) exposure is a risk factor in laryngeal cancer. HPV DNA transforms the moist membranes of epithelial cells (cervix, anus, mouth, and throat). The juvenile type is related to vertical transmission and the adult type to orogenital contact. HPV subtype 16 accounts for the majority of oral tumors, oropharynx cancers, and laryngeal cases.

280. Which of the following is considered an abnormal result on a Weber test?

Lateralization to one ear The Weber test is a quick screening test for hearing. It can detect unilateral conductive hearing loss (middle ear hearing loss) and unilateral sensorineural hearing loss (inner ear hearing loss). In the Weber test, a vibrating tuning fork is placed in the middle of the forehead, above the upper lip, under the nose, over the teeth, or on top of the head equidistant from the patient's ears on top of thin skin in contact with the bone. In a normal patient, the Weber tuning fork sound is heard equally loud in both ears, with no one ear hearing the sound louder than the other (lateralization). In a patient with hearing loss (otitis media, cerumenosis), the Weber tuning fork sound is heard louder in one ear (lateralization) than the other (the "bad" ear).

A 60-year-old baker presents to your clinic, complaining of increasing shortness of breath and nonproductive cough over the last month. She feels like she can't do as much activity as she used to do without becoming tired. She even has to sleep upright in her recliner at night to be able to breathe comfortably. She denies any chest pain, nausea, or sweating. Her past medical history is significant for high blood pressure and coronary artery disease. She had a hysterectomy in her 40s for heavy vaginal bleeding. She is married and is retiring from the local bakery soon. She denies any tobacco, alcohol, or drug use. Her mother died of a stroke and her father died from prostate cancer. She denies any recent upper respiratory illness, and she has had no other symptoms. On examination she is in no acute distress. Her blood pressure is 160/100 and her pulse is 100. She is afebrile and her respiratory rate is 16. With auscultation she has distant air sounds and she has late inspiratory crackles in both lower lobes. On cardiac examination the S1 and S2 are distant and an S3 is heard over the apex. What disorder of the chest best describes her symptoms?

Left-sided heart failure In left-sided heart failure, fluid starts "backing up" into the lungs because the heart is unable to handle the volume. The excess fluid collects in the dependent areas, causing crackles in the bases of the lower lobes. Sitting up allows patients to breathe easier. The two main causes are chronic high blood pressure and coronary artery disease, which lead to myocardial ischemia and decreased contractility of the heart.

140. Which of the following findings is associated with the chronic use of chewing tobacco?

Leukoplakia and oral cancer The chronic use of tobacco increases the risk of oral cancer and leukoplakia. Cheilosis is skin fissures/maceration in the corner of the mouth, most commonly caused by anemia, bacterial infection, vitamin deficiencies, or oversalivation.

You feel a small mass that you think is a lymph node. It is mobile in both the up-and-down and side-to-side directions. Which of the following is most likely?

Lymph node A useful maneuver for discerning lymph nodes from other masses in the neck is to check for their mobility in all directions. Many other masses are mobile in only two directions. Cancerous masses may also be "fixed," or immobile.

A 32-year-old white female comes to your clinic, complaining of overwhelming sadness. She says for the past 2 months she has had crying episodes, difficulty sleeping, and problems with overeating. She says she used to go out with her friends from work but now she just wants to go home and be by herself. She also thinks that her work productivity has been dropping because she just is too tired to care or concentrate. She denies any feelings of guilt or any suicidal ideation. She states that she has never felt this way in the past. She denies any recent illness or injuries. Her past medical history consists of an appendectomy when she was a teenager; otherwise, she has been healthy. She is single and works as a clerk in a medical office. She denies tobacco, alcohol, or illegal drug use. Her mother has high blood pressure and her father has had a history of mental illness. On examination you see a woman appearing her stated age who seems quite sad. Her facial expression does not change while you talk to her and she makes little eye contact. She speaks so softly you cannot always understand her. Her thought processes and content seem unremarkable. What type of mood disorder do you think she has?

Major depressive episode

A 23-year-old ticket agent is brought in by her husband because he is concerned about her recent behavior. He states that for the last 2 weeks she has been completely out of control. He says that she hasn't showered in days, stays awake most of the night cleaning their apartment, and has run up over $1,000 on their credit cards. While he is talking, the patient interrupts him frequently and declares this is all untrue and she has never been so happy and fulfilled in her whole life. She speaks very quickly, changing the subject often. After a longer than normal interview you find out she has had no recent illnesses or injuries. Her past medical history is unremarkable. Both her parents are healthy but the husband has heard rumors about an aunt with similar symptoms. She and her husband have no children. She smokes one pack of cigarettes a day (although she has been chain-smoking in the last 2 weeks), drinks four to six drinks a week, and smokes marijuana occasionally. On examination she is very loud and outspoken. Her physical examination is unremarkable. Which mood disorder does she most likely have?

Manic episode

A 50-year-old body builder is upset by a letter of denial from his life insurance company. He is very lean but has gained 2 pounds over the past 6 months. You personally performed his health assessment and found no problems whatsoever. He says he is classified as "high risk" because of obesity. What should you do next?

Measure his waist

A patient with longstanding COPD was told by another practitioner that his liver was enlarged and this needed to be assessed. Which of the following would be reasonable to do next? A) Percuss the lower border of the liver B) Measure the span of the liver C) Order a hepatitis panel D) Obtain an ultrasound of the liver

Measure the span of the liver In this patient, measuring the span of the liver saved the patient an involved workup, because it was normal. His history of COPD is consistent with flattening of the diaphragms, which pushed the liver edge down while the actual size of the liver remained the same. Percussing the lower border of the liver alone caused this referral, because it was assumed that the liver was enlarged.

542. A 65-year-old man presents to the clinic complaining of random recurrent episodes of dizziness with nausea. The patient describes it as the sensation of the room moving or of the room spinning. It is worsened by sudden head movement. During the episodes, he becomes very nauseated. He also has tinnitus with hearing loss in his right ear. The patient has type 2 diabetes and is on a prescription of metformin 500 mg PO BID and an angiotensin-converting enzyme (ACE) inhibitor. The blood glucose level during his visit is 80 mg/dL. Which of the following conditions is most likely?

Meniere's Disease The classic triad of symptoms of Ménière's disease is episodic vertigo, tinnitus, and sensorineural hearing loss (low frequency). Tinnitus is usually low pitch (like listening to a conch shell). One may have a strong sensation of ear fullness. The condition can resolve spontaneously or may be chronic. Pathologic lesion in the middle ear is called endolymphatic hydrops. Vasovagal syncope does not cause hearing loss or tinnitus, nor is it episodic. Hypoglycemia is not associated with episodic vertigo, tinnitus, and hearing loss.

A 55-year-old bank teller comes to your office for persistent episodes of dizziness. The first episode started suddenly and lasted 3 to 4 hours. He experienced a lot of nausea with vomiting; the episode resolved spontaneously. He has had five episodes in the past 11⁄2 weeks. He does note some tinnitus that comes and goes. Upon physical examination, you note that he has a normal gait. The Weber localizes to the right side and the air conduction is equal to the bone conduction in the right ear. Nystagmus is present. Based on this description, what is the most likely diagnosis?

Meniere's disease Ménière's disease is characterized by sudden onset of vertiginous episodes that last several hours to a day or more, then spontaneously resolve; the episodes then recur. On physical examination, sensorineural hearing loss is present. The patient does complain of tinnitus.

331. All of the following are clinical eye findings found in some patients with chronic uncontrolled hypertension. Which is not associated with this disorder?

Microaneurysms Keith Wagener Barker (KWB) grades for uncontrolled hypertension are as follows: Grade 1—generalized arteriolar constriction seen as "silver wiring" and vascular tortuousities; grade 2—grade 1 plus irregularly located, tight constrictions known as AV nicking or AV nipping; grade 3—grade 2 plus cotton wool spots and flame-shaped hemorrhages; and grade 4—grade 3, but with swelling of the optic disc (papilledema). Microaneurysms occur with diabetic retinopathy.

468. Three of the following are eye findings associated with chronic uncontrolled hypertension. Which one of the following is associated with diabetic retinopathy?

Microaneurysms Microaneurysms are seen with diabetic retinopathy. Arteriovenous (AV) nicking, copper wire arterioles, and flame hemorrhages are seen with uncontrolled hypertension.

A 38-year-old accountant comes to your clinic for evaluation of a headache. The throbbing sensation is located in the right temporal region and is an 8 on a scale of 1 to 10. It started a few hours ago, and she has noted nausea with sensitivity to light; she has had headaches like this in the past, usually less than one per week, but not as severe. She does not know of any inciting factors. There has been no change in the frequency of her headaches. She usually takes an over-the-counter analgesic and this results in resolution of the headache. Based on this description, what is the most likely diagnosis of the type of headache?

Migraine This is a description of a common migraine (no aura). Distinctive features of a migraine include phonophobia and photophobia, nausea, resolution with sleep, and unilateral distribution. Only some of these features may be present.

255. A 13-year-old boy's peak expiratory flow results indicate 60% to 80% of the predicted range. How would you classify his asthma?

Moderate persistent asthma The Global Initiative for Asthma has four clinical classifications of severity: intermittent (normal FEV1 between exacerbations, FEV1 >80%), mild persistent (FEV1 >80%), moderate persistent (FEV1 60%- 80%), and severe persistent (FEV1 <60%). Daily symptoms with more than one nighttime episode of symptoms per week, 60% to 80% FEV1, and greater than 30% FEV1 variability and need for short-acting beta-2 agonist for symptom control, are classified as moderate persistent severity in patients older than 12 years of age.

63M presents to establish care at new PCP to discuss issues with pain & fatigue. Clinician conducting visit begins with general historical questions but quickly becomes suspicious that the pt is suffering from decompensated HF. When pt mentions he has had vague CP since last night, clinician feels the focus must be redirected to potentially emergent condition, which of the following interview techniques is most appropriate?

Moving from open-ended to focused questions

65yo retired pilot visits clinical DT recurrent HA, dizziness of recent onset (previous 2 weeks), occassional numbness on left side, which of the following systems/regions should be examined?

Nervous system

Mr. Garcia comes to your office for a rash on his chest associated with a burning pain. Even a light touch causes this burning sensation to worsen. On examination, you note a rash with small blisters (vesicles) on a background of reddened skin. The rash overlies an entire rib on his right side. What type of pain is this?

Neuropathic pain

When using an interpreter to facilitate an interview, where should the interpreter be positioned?

Next to the patient, so the examiner can maintain eye contact and observe the nonverbal cues of the patient

Is the following information subjective or objective? Mrs. M has a RR of 32 and PR 120

Objective

47 yo fitness trainer comes for skin dryness, night sweats, irregular menstrual periods, first contact w pt. notes "my sex life has really gone downhill lately" is considering divorcing husband of 20 years, states "he's not a bad guy I just think that I can do better", how should the PA proceed?

Obtain a menstrual hx for previous 6 mos

39yo nurse well-established pt co irregular menses & pelvis pain, having trouble sleeping, wants a "sleeping pill", thinking of leaving her job, what is the best next step?

Obtain more complex description of problems

You are conducting a pupillary examination on a 34-year-old man. You note that both pupils dilate slightly. Both are noted to constrict briskly when the light is placed on the right eye. What is the most likely problem?

Optic nerve damage on the left Because both pupils can constrict, efferent nerve damage is unlikely. When the light is placed on the left eye, neither a direct nor a consensual response is seen. This indicates that the left eye is not perceiving incoming light.

Sudden, painful unilateral loss of vision may be caused by which of the following conditions?

Optic neuritis In multiple sclerosis, sudden painful loss of vision may accompany optic neuritis. The other conditions are usually painless.

A patient with hearing loss by whisper test is further examined with a tuning fork, using the Weber and Rinne maneuvers. The abnormal results are as follows: bone conduction is greater than air on the left, and the patient hears the sound of the tuning fork better on the left. Which of the following is most likely?

Otosclerosis of the left ear The above pattern is consistent with a conductive loss on the left side. Causes would include: foreign body, otitis media, perforation, and otosclerosis of the involved side.

A 24-year-old secretary comes to your clinic, complaining of difficulty sleeping, severe nightmares, and irritability. She states it all began 6 months ago when she went to a fast food restaurant at midnight. While she was waiting in her car a man entered through the passenger door and put a gun to her head. He had her drive to a remote area, where he took her money and threatened to kill her. When the gun jammed he panicked and ran off. Ever since this occurred the patient has been having these symptoms. She states she jumps at every noise and refuses to drive at night. She states her anxiety has had such a marked influence on her job performance she is afraid she will be fired. She denies any recent illnesses or injuries. Her past medical history is unremarkable. On examination you find a nervous woman appearing her stated age. Her physical examination is unremarkable. You recommend medication and counseling. What anxiety disorder to you think this young woman has?

PTSD

136. During the eye exam of a 50-year-old hypertensive patient who is complaining of an onset of a severe headache, you find that the borders of the disc margins on both eyes are blurred. What is the name of this clinical finding?

Papilledema The funduscopic examination visualizes vessels and assesses intracranial tension and is recommended in new-onset headaches. Papilledema is optic disc swelling cause by increased intracranial pressure. The swelling is usually bilateral. Signs include venous engorgement, loss of venous pulsation, hemorrhages over and/or adjacent to the optic disc, blurring of optic margins, and elevation of the optic disc. On visual field exam there may be an enlarged blind spot.

29F bookseller presents to PCP with complaint of depression, she has 2 young children at home & expresses worry about her ability to support them, when asked about role of her partner (who also receives care at clinic, visited clinician with pt before), pts voice wavers & becomes lower in pitch & replies, "he's fine". clinician notes change in tone & queries further, only to find out recent financial stressors have reactivated the partner's prior abusive behavior, which of the following best describes form of communication that helped clinician identify this issue?

Paralanguage, consists of tone, pacing and volume of speech that provides indirect verbal clues to the pts mood, intent, etc.

A 73-year-old nurse comes to your office for evaluation of new onset of tremors. She is not on any medications and does not take herbs or supplements. She has no chronic medical conditions. She does not smoke or drink alcohol. She walks into the examination room with slow movements and shuffling steps. She has decreased facial mobility and a blunt expression, without any changes in hair distribution on her face. Based on this description, what is the most likely reason for the patient's symptoms?

Parkinson's disease This is a typical description for a patient with Parkinson's disease. Facial mobility is decreased, which results in a blunt expression—a "masked" appearance. The patient also has decreased blinking and a characteristic stare with an upward gaze. In combination with the findings of slow movements and a shuffling gait, the diagnosis of Parkinson's is almost clinched.

311. Acute bronchitis is best characterized by:

Paroxysms of coughing that are dry or produce mucoid sputum A cough is the main symptom of acute bronchitis. It may be dry at first (does not produce mucus) and after a few days may bring up mucus from the lungs (productive cough). The mucus may be clear, yellow, or green. Small streaks of blood may be present.

Depression of lower part of sternum:

Pectus excavatum Congenital abnormality in which inferior part of sternum displaced inward Barrel chest: Increased AP diameter, often in COPD Flail chest: injured area of flail chest moves inward with inspiration & moves outward with expiration Pigeon chest: pectus carinatum, congenital abnormality in which sternum is displaced anteriorly Thoracic kyphoscoliosis: abnormal spinal curvatures and vertebral rotation, visible posteriorly (rather than anteriorly)

A 36-year-old teacher presents to your clinic, complaining of sharp, knifelike pain on the left side of her chest for the last 2 days. Breathing and lying down make the pain worse, while sitting forward helps her pain. Tylenol and ibuprofen have not helped. Her pain does not radiate to any other area. She denies any upper respiratory or gastrointestinal symptoms. Her past medical history consists of systemic lupus. She is divorced and has one child. She denies any tobacco, alcohol, or drug use. Her mother has hypothyroidism and her father has high blood pressure. On examination you find her to be distressed, leaning over and holding her left arm and hand to her left chest. Her blood pressure is 130/70, her respirations are 12, and her pulse is 90. On auscultation her lung fields have normal breath sounds with no rhonchi, wheezes, or crackles. Percussion and palpation are unremarkable. Auscultation of the heart has an S1 and S2 with no S3 or S4. A scratching noise is heard at the lower left sternal border, coincident with systole; leaning forward relieves some of her pain. She is nontender with palpation of the chest wall. What disorder of the chest best describes this disorder?

Pericarditis The pain from pericarditis is usually sharp and knifelike and is located over the left side of the chest. Change of position, breathing, and coughing often make the pain worse, whereas leaning forward improves the pain. Pericarditis is often seen in rheumatologic diseases such as systemic lupus and in patients with chronic kidney disease. Patients also experience this after a myocardial infarction. You can read more about Dressler's syndrome.

425. A 35-year-old man has a history of an upper respiratory viral infection 4 weeks ago. He reports that he started feeling short of breath and now complains of sharp pain in the middle of his chest that seems to worsen when he lies down. The patient's physical exam is within normal limits with the exception of a precordial rub on auscultation. The most likely diagnosis would be:

Pericarditis Pericarditis is inflammation of the sac around the heart. Common signs and symptoms include chest pain over the center/left side of the chest; shortness of breath, especially with lying down; low-grade fever; weakness; fatigue; dry cough; and abdominal or leg swelling. Pericardial rub may be auscultated.

177. A test called the visual fields by confrontation is used to evaluate for:

Peripheral vision The visual fields by confrontation test is used to evaluate peripheral vision. The Snellen chart is used to measure central distance vision. A tonometer is used to assess for glaucoma. The ophthalmoscope is used to assess for cataracts.

In obtaining a history, you note that a patient uses the word "largely" repeatedly, to the point of being a distraction to your task. Which word best describes this speech pattern?

Perseveration Perseveration is the repetition of words or ideas. Echolalia differs in that the patient repeats what is said to him. Clanging is the repetition of the same sounds in different words. Confabulation is making up a story in response to a question. This is sometimes seen in chronic alcohol use with Korsakoff's syndrome.

The following information is recorded in the health history, "the patient completed 8th grade, currently lives with his wife & 2 children, works on old cars on weekend, works in glass factory during the week." Which category?

Personal and social history

A 25-year-old accountant presents to your clinic, complaining of intermittent lower right-sided chest pain for several days. He describes it as knifelike and states it only lasts for 3 to 5 seconds, taking his breath away. He states he feels like he has to breathe shallowly to keep it from recurring. The only thing that makes it better is lying quietly on his right side. It is much worse when he takes a deep breath. He has taken some Tylenol and put a heating pad on his side but neither has helped. He remembers that 2 weeks ago he had an upper respiratory infection with a severe hacking cough. He denies any recent trauma. His past medical history is unremarkable. His parents and siblings are in good health. He has recently married, and his wife has a baby due in 2 months. He denies any smoking or illegal drug use. He drinks two to three beers once a month. He states that he eats a healthy diet and runs regularly, but not since his recent illness. He denies any cardiac, gastrointestinal, or musculoskeletal symptoms. On examination he is lying on his right side but appears quite comfortable. His temperature, blood pressure, pulse, and respirations are unremarkable. His chest has normal breath sounds on auscultation. Percussion of the chest is unremarkable. During palpation the ribs are nontender. What disorder of the chest best describes his symptoms?

Pleural pain This pain is sharp and knifelike and occurs over the affected area of pleura. Breathing deeply usually makes the pain worse, whereas lying quietly on the affected side makes the pain better. Pleurisy often occurs from inflammation due to an infection, neoplasm, or autoimmune disease.

A 47-year-old receptionist comes to your office, complaining of fever, shortness of breath, and a productive cough with golden sputum. She says she had a cold last week and her symptoms have only gotten worse, despite using over-the-counter cold remedies. She denies any weight gain, weight loss, or cardiac or gastrointestinal symptoms. Her past medical history includes type 2 diabetes for 5 years and high cholesterol. She takes an oral medication for both diseases. She has had no surgeries. She denies tobacco, alcohol, or drug use. Her mother has diabetes and high blood pressure. Her father passed away from colon cancer. On examination you see a middle-aged woman appearing her stated age. She looks ill and her temperature is elevated, at 101. Her blood pressure and pulse are unremarkable. Her head, eyes, ears, nose, and throat examinations are unremarkable except for edema of the nasal turbinates. On auscultation she has decreased air movement, and coarse crackles are heard over the left lower lobe. There is dullness on percussion, increased fremitus during palpation, and egophony and whispered pectoriloquy on auscultation. What disorder of the thorax or lung best describes her symptoms?

Pneumonia Pneumonia is usually associated with dyspnea, cough, and fever. On auscultation there can be coarse or fine crackles heard over the affected lobe. Percussion over the affected area is dull and there is often an increase in fremitus. Egophony and pectoriloquy are heard because of increased transmission of high-pitched components of sounds. These higher frequencies are usually filtered out by the multiple air-filled chambers of the alveoli.

A college student presents with a sore throat, fever, and fatigue for several days. You notice exudates on her enlarged tonsils. You do a careful lymphatic examination and notice some scattered small, mobile lymph nodes just behind her sternocleidomastoid muscles bilaterally. What group of nodes is this?

Posterior cervical The group of nodes posterior to the sternocleidomastoid muscle is the posterior cervical chain. These are common in mononucleosis.

228. A 10-year-old boy who was recently accepted onto his school's soccer team has a history of exercise-induced asthma. He wants to know when he should use his albuterol inhaler. The nurse practitioner would advise the patient to:

Premedicate 10-15 mins before starting exercise Exercise-induced asthma is best controlled by using the Proventil inhaler (bronchodilator) approximately 10 to 15 minutes before exercise, to prevent vasospasm of the bronchioles and shortness of breath with exercise. The effects of these bronchodilators usually last approximately 4 hours. They also work quickly to open up the bronchioles if an acute attack/shortness of breath occurs.

657. A 6-year-old girl who attends preschool part time is brought to the clinic by her mother as a walk-in patient. The mother reports that her daughter has recently begun swim lessons. The symptoms began as redness on the left eye and spread to the second eye within 2 days. The child's eyes are watery and crusted in the morning when she wakes up. Her vital signs are temperature of 98.8°F, pulse of 90 beats/min, and respirations of 16 breaths/min. The eye exam reveals bilateral injected conjunctiva. When the lower eyelid is examined, the nurse practitioner notes that it is pink with a cobblestone appearance. There is ipsilateral preauricular adenopathy. All of the following treatment measures are appropriate except:

Prescribe ophthalmic topical antibiotic eye drops, two to three drops to be applied in each eye QID for 7 days Treatment for viral keratoconjunctivitis (pink eye) is symptomatic. Cold compresses and slightly chilled artificial tears may help with the itching. Educate the parent and/or child to avoid touching the eyes with hands, avoid sharing towels, to perform frequent handwashing, and use tissues if touching the eyes. Viral keratoconjunctivitis is usually caused by an adenovirus (but other viruses can also cause it). It is contagious for 10 to 12 days after onset of symptoms and is a self-limiting condition. It can be transmitted through swimming pools, fomites, and by hands. Children should not attend school until symptoms resolve

The following information is recorded in the healthy history, pt has had abdominal pain for 1 week, pain lasts for 30 mins at a time, comes & goes, 7-9/10 pain, accompanied by NV, located mid-epigastric. Which of these categories does it belong to?

Present Illness

You are at your family reunion playing football when your uncle takes a hit to his right lateral thorax and is in pain. He asks you if you think he has a rib fracture. You are in a very remote area. What would your next step be? A) Call a medevac helicopter B) Drive him to the city (4 hours away) C) Press on his sternum and spine simultaneously D) Examine him for tenderness over the injured area

Press on his sternum and spine simultaneously The area involved in the injury will of course be tender. If you press in an area remote to the injury, but over the same bone which may be involved, you can produce tenderness at the site of injury. This would indicate that there may be a fracture at the lateral ribs. Fortunately, this maneuver did not reproduce pain remotely, and your uncle simply sat on the sidelines for the rest of the game.

42F presents with fatigue associated with 40lb weight gain over past 2 yrs, always struggled with weight but has continued to gain despite various attempts at diet & exercise regimens, inquires if she might be candidate for gastric bypass surgery. In evaluating pts who are overweight, which of the following best defines obesity in medical terms?

Pt with BMI > 30

A 67-year-old lawyer comes to your clinic for an annual examination. He denies any history of eye trauma. He denies any visual changes. You inspect his eyes and find a triangular thickening of the bulbar conjunctiva across the outer surface of the cornea. He has a normal pupillary reaction to light and accommodation. Based on this description, what is the most likely diagnosis?

Pterygium A pterygium is a triangular thickening of the bulbar conjunctiva that grows slowly across the outer surface of the cornea, usually from the nasal side. Reddening may occur, and it may interfere with vision as it encroaches on the pupil. Otherwise, treatment is unnecessary.

119. During a routine physical exam of an elderly woman, a triangular thickening of the bulbar conjunctiva on the temporal side is noted to be encroaching on the cornea. She denies any eye pain or visual changes. Which of the following is most likely?

Pterygium Pterygium is a triangular growth on the white part of the eye that also extends onto the cornea. Corneal arcus is a white- to greyish-colored ring around the edge of the cornea of both eyes. Chalazion is a stye in the eye that may cause pain and swelling. Pinguecula is a benign growth on the conjunctiva caused by the degeneration of its collagen fibers. Thick, yellow fibers may be seen.

A patient is describing a very personal part of her history very quickly and in great detail. How should you react to this?

Push away from the keyboard or put down your pen & listen

You arrive at the bedside of an elderly woman who has had a stroke, affecting her entire right side. She cannot speak (aphasia). You are supposed to examine her. You notice that the last examiner left her socks at the bottom of the bed, and although sensitive areas are covered by a sheet, the blanket is heaped by her feet at the bottom of the bed. What would you do next?

Put her socks back on & cover her completely before beginning the evaluation

A 25-year-old type 1 diabetic clerk presents to the emergency room with shortness of breath and states that his blood sugar was 605 at home. You diagnose the patient with diabetic ketoacidosis. What is the expected pattern of breathing?

Rapid and deep

A 15-year-old high school sophomore and her mother come to your clinic because the mother is concerned about her daughter's weight. You measure her daughter's height and weight and obtain a BMI of 19.5 kg/m2. Based on this information, which of the following is appropriate?

Reassure the mother that this is a normal bodyweight Normal BMI is 18.5-24.9

You are interviewing an elderly woman in the ambulatory setting and trying to get more information about her urinary symptoms. Which of the following techniques is not a component of adaptive questioning?

Reassuring the patient that the urinary symptoms are benign and that she doesn't need to worry about it being a sign of cancer

36F air traffic controller presents to PCP for routine visit 3 mos after losing her spouse to a lengthy battle with neurodegenerative disease, denies any psych s/s on ROS, states she has slept better in last month than in previous years, endorses healthy support system, including extended family of her deceased spouse whom she is still close with, becomes wistful, briefly tearful when speaking of plans they had when they first married that they never fulfilled, changes subject rapidly to whether her pap smear is due. Which is an example of an empathetic response?

Recognizing pts emotions by asking or confirming how she feels about the event

74M being seen bc of 1 day hx of painful right eye. also mentions he has blurred vision in that eye. he thought something had blown into his eye, but after flushing it out, the pain and blurred vision remains. what is the best course of action?

Refer to an ophthalmologist emergently with the possibility of corneal ulcer, uveitis or acute glaucoma unilateral, painful eye may be due to acute glaucoma or corneal pathology (foreign body, ulcer, uveitis)

a 55yo air traffic control agent reports his home BP log to clinic after he was dx with HTN at a prior visit, he notes that he consistently measures within the normal range at home, but seems to fall outside the normal range every time he comes to the clinic. Which of the following BP measurements is considered to be most accurate (reflecting "true" BP)?

Regular ambulatory monitoring recorded outside of the office setting By be influenced by stress of presenting to a clinician "white coat HTN"

Ms. Wright comes to your office, complaining of palpitations. While checking her pulse you notice an irregular rhythm. When you listen to her heart, every fourth beat sounds different. It sounds like a triplet rather than the usual "lub dup." How would you document your examination?

Regularly irregular rhythm

A sudden, painless unilateral vision loss may be caused by which of the following?

Retinal detachment Corneal ulcer, acute glaucoma, and uveitis are almost always accompanied by pain. Retinal detachment is generally painless, as is chronic glaucoma.

144. While checking for the red reflex on a 3-year-old boy during a well-child visit, the nurse practitioner notes a white reflection on the child's left pupil. Which of the following conditions should be ruled out?

Retinoblastoma of the left eye Retinoblastoma is a congenital tumor of the retina. It usually affects only one eye (rarely both eyes are involved). During infancy, the tumor is a small size and it continues to grow with the child. This rare cancer is diagnosed by noting a pupil that appears white or has white spots on it. One or both eyes may be affected. It is often first noted in photographs, because a white glow is present in the eye instead of the usual "red eye" that results from the flash

The following information is recorded in health history, "patient denies CP, palpitations, orthopnea, paroxysmal nocturnal dyspnea", which category?

Review of systems

On visual confrontation testing, a stroke patient is unable to see your fingers on his entire right side with either eye covered. Which of the following terms would describe this finding?

Right homonymous hemianopsia Because the right visual field in both eyes is affected, this is a right homonymous hemianopsia. A bitemporal hemianopsia refers to loss of both lateral visual fields. A right temporal hemianopsia is unilateral and binasal hemianopsia is the loss of the nasal visual fields bilaterally.

A 19-year-old college student, Todd, is brought to your clinic by his mother. She is concerned that there is something seriously wrong with him. She states for the past 6 months his behavior has become peculiar and he has flunked out of college. Todd denies any recent illness or injuries. His past medical history is remarkable only for a broken foot. His parents are both healthy. He has a paternal uncle who had similar symptoms in college. The patient admits to smoking cigarettes and drinking alcohol. He also admits to marijuana use but none in the last week. He denies using any other substances. He denies any feelings of depression or anxiety. While speaking with Todd and his mother you do a complete physical examination, which is essentially normal. When you question him on how he is feeling, he says that he is very worried that Microsoft has stolen his software for creating a better browser. He tells you he has seen a black van in his neighborhood at night and he is sure that it is full of computer tech workers stealing his work through special gamma waves. You ask him why he believes they are trying to steal his programs. He replies that the technicians have been telepathing their intents directly into his head. He says he hears these conversations at night so he knows this is happening. Todd's mother then tells you, "See, I told you . . . he's crazy. What do I do about it?" While arranging for a psychiatry consult, what psychotic disorder do you think Todd has?

Schizophrenia

Which of the following is a symptom involving the eye?

Scotomas Specks in the vision or areas where pt cannot see

Jason is a 41-year-old electrician who presents to the clinic for evaluation of shortness of breath. The shortness of breath occurs with exertion and improves with rest. It has been going on for several months and initially occurred only a couple of times a day with strenuous exertion; however, it has started to occur with minimal exertion and is happening more than a dozen times per day. The shortness of breath lasts for less than 5 minutes at a time. He has no cough, chest pressure, chest pain, swelling in his feet, palpitations, orthopnea, or paroxysmal nocturnal dyspnea.

Severity

A patient is assigned a visual acuity of 20/100 in her left eye. Which of the following is true?

She can see at 20ft what a normal person could see at 100ft The denominator of an acuity score represents the line on the chart the patient can read. In the example above, the patient could read the larger letters corresponding with what a normal person could see at 100 feet.

Jenny is one of your favorite patients who usually shares a joke with you and is nattily dressed. Today she is dressed in old jeans, lacks makeup, and avoids eye contact. To what do you attribute these changes?

She is depressed

Mrs. R. is a 92-year-old retired teacher who comes to your clinic accompanied by her daughter. You ask Mrs. R. why she came to your clinic today. She looks at her daughter and doesn't say anything in response to your question. This is an example of which type of challenging patient?

Silent patient

705. Transillumination is useful in helping to diagnose which of the following conditions?

Sinusitis and hydrocele Transillumination is used for evaluation of the frontal and maxillary sinus as well as for a hydrocele. Because light is able to pass through the delicate skin covering the hollow sinus cavities, a light source held against the upper cheek will produce a red dot on the palate if the sinuses are normal (filled with air rather than obstructed). The transillumination test is used to differentiate a hydrocele from hernia—an illuminated scrotum will show the testicle in the center surrounded by water in the hydrocele.

Which of the following occurs in respiratory distress? A) Speaking in sentences of 10-20 words B) Skin between the ribs moves inward with inspiration C) Neck muscles are relaxed D) Patient torso leans posteriorly

Skin btx ribs moves inward with inspiration This description is consistent with retractions that occur with respiratory distress. Other features include speaking in short sentences, use of accessory muscles, leaning forward to gain mechanical advantage for the diaphragm, and pursed lip breathing, in which the patient exhales against his lips, which are pressed together.

A 49-year-old truck driver comes to the emergency room for shortness of breath and swelling in his ankles. He is diagnosed with congestive heart failure and admitted to the hospital. You are the student assigned to do the patient's complete history and physical examination. When you palpate the pulse, what do you expect to feel?

Small amplitude, weak

A 26-year-old violinist comes to your clinic, complaining of anxiety. He is a first chair violinist in the local symphony orchestra and has started having symptoms during performances, such as sweating, shaking, and hyperventilating. It has gotten so bad that he has thought about giving up his first chair status so he does not have to play the solo during one of the movements. He says that he never has these symptoms during rehearsals or when he is practicing. He denies having any of these symptoms at any other time. His past medical history is unremarkable. He denies any tobacco use, drug use, or alcohol abuse. His parents are both healthy. On examination you see a young man who appears worried. His vital signs and physical examination are unremarkable. What type of anxiety disorder best describes his situation?

Social phobia

A 17-year-old high school senior presents to your clinic in acute respiratory distress. Between shallow breaths he states he was at home finishing his homework when he suddenly began having right-sided chest pain and severe shortness of breath. He denies any recent traumas or illnesses. His past medical history is unremarkable. He doesn't smoke but drinks several beers on the weekend. He has tried marijuana several times but denies any other illegal drugs. He is an honors student and is on the basketball team. His parents are both in good health. He denies any recent weight gain, weight loss, fever, or night sweats. On examination you see a tall, thin young man in obvious distress. He is diaphoretic and is breathing at a rate of 35 breaths per minute. On auscultation you hear no breath sounds on the right side of his superior chest wall. On percussion he is hyperresonant over the right upper lobe. With palpation he has absent fremitus over the right upper lobe. What disorder of the thorax or lung best describes his symptoms?

Spontaneous pneumothorax Spontaneous pneumothorax occurs suddenly, causing severe dyspnea and chest pain on the affected side. It is more common in thin young males. On auscultation of the affected side there will be no breath sounds and on percussion there is hyperresonance or tympany. There will be an absence of fremitus to palpation. Given this young man's habitus and pneumothorax, you may consider looking for features of Marfan's syndrome. Read more about this condition.

158. The cover/uncover test is a screening tool for:

Strabismus The cover/uncover test screens for strabismus. Color blindness is evaluated by using the Ishihara tool. To use the Ishihara chart, the child must be familiar with reading numbers and be able to follow instructions. Visual acuity (distance vision) is evaluated using the Snellen chart. Cataracts are screened by using the red reflex test. Use the direct ophthalmoscope (set at "0") and shine the light into both the eyes at about 18 inches away from the patient. Instruct the patient to stare in one direction forward (and avoid moving the eyes). Turn off the room light. In someone with advanced cataracts, the red reflex will show a white reflection instead of the normal orange to red color.

Which of the following is consistent with good percussion technique? A) Allow all of the fingers to touch the chest while performing percussion. B) Maintain a stiff wrist and hand. C) Leave the plexor finger on the pleximeter after each strike. D) Strike the pleximeter over the distal interphalangeal joint.

Strike the pleximeter over the distal interphalangeal joint Percussion takes practice to master. Most struggle initially with keeping the wrist and hand relaxed. Other challenges include removing the plexor quickly and keeping the other fingers off the chest wall. These can dampen the sound you are trying to obtain. The ideal target for the plexor is the distal interphalangeal joint.

Good example for good technique for percussion

Strike using tip of third finger Strike distal interphalangeal joint Only third finger pressed against chest Wrist: brisk yet relaxed

395. While performing a routine physical exam on a 60-year-old man, the nurse practitioner notices a soft bruit over the carotid area on the left side of the neck. The patient has a history of hypertension. The patient is at higher risk for:

Stroke and coronary artery disease Carotid stenosis puts the patient at risk for stroke and coronary artery disease. A bruit is a murmur heard over the carotid artery in the neck, suggesting arterial narrowing and atherosclerosis. It may increase risk of cerebrovascular disease. Bruits at the bifurcation of the common carotid artery are best heard high up under the angle of the jaw. At this level the common carotid artery bifurcates and gives rise to its internal branch. If one hears a bruit only in the base of the neck, or along the course of the common carotid artery, it is referred to as diffuse. Diffuse bruits are not a very specific indicator of internal carotid artery disease. Bruits heard only at the bifurcation are more specific for internal carotid artery origin stenosis.

A 15-year-old high school sophomore presents to the emergency room with his mother for evaluation of an area of blood in the left eye. He denies trauma or injury but has been coughing forcefully with a recent cold. He denies visual disturbances, eye pain, or discharge from the eye. On physical examination, the pupils are equal, round, and reactive to light, with a visual acuity of 20/20 in each eye and 20/20 bilaterally. There is a homogeneous, sharply demarcated area at the lateral aspect of the base of the left eye. The cornea is clear. Based on this description, what is the most likely diagnosis?

Subconjunctival hemorrhage A subconjunctival hemorrhage is a leakage of blood outside of the vessels, which produces a homogenous, sharply demarcated bright red area; it fades over several days, turning yellow, then disappears. There is no associated eye pain, ocular discharge, or changes in visual acuity; the cornea is clear. Many times it is associated with severe cough, choking, or vomiting, which increase venous pressure. It is rarely caused by a serious condition, so reassurance is usually the only treatment necessary.

54. This case is followed by questions and multiple-choice answers specific to the case. A 70- year-old male patient complains of a bright-red-colored spot that has been present in his left eye for 2 days. He denies eye pain, visual changes, or headaches. He has a new-onset cough from a recent viral upper respiratory infection. The only medicine he is taking is Bayer aspirin one tablet a day. Which of the following is most likely?

Subconjunctival hemorrhage Bright-red blood in a sharply defined area surrounded by normal-appearing conjunctiva indicates subconjunctival hemorrhage. The blood stays red because of direct diffusion of oxygen through the conjunctiva. Risk factors include diabetes, hypertension, illnesses that cause severe coughing or sneezing, blood-thinning medications and aspirin, and herbal supplements such as ginkgo.

Is the following information subjective or objective? Mrs. M has SOB that has persisted for the past 10 days, it is worse with activity & relieved by rest.

Subjective Information given by pt about circumstances of chief complaint, does not represent objective observation

Which of the following complaints/findings is considered to be a pt identifier for mental health screening?

Substance abuse

The following information is best placed in which category? "The pt has had 3 c-sections"

Surgeries

A 29-year-old computer programmer comes to your office for evaluation of a headache. The tightening sensation is located all over the head and is of moderate intensity. It used to last minutes, but this time it has lasted for 5 days. He denies photophobia and nausea. He spends several hours each day at a computer monitor/keyboard. He has tried over-the-counter medication; it has dulled the pain but not taken it away. Based on this description, what is your most likely diagnosis?

Tension

62yo former tennis pro obtained a home BP cuff after an office measurement revealed that his BP fell into the hypertensive range, at fu, questions accuracy of clinician's BP cuff & veracity of dx of HTN. which of the following is true regarding BP recorded in practitioner's office vs. values obtained in ambulatory setting?

The accepted normal values for BP are lower for ambulatory measurements compared with office measurements. BP < 140/90 is considered normal for office measurement, outside < 135/85

A first semester physician assistant students reports to his supervisor that he has trouble determining the DBP, on manual BP, which of the following provides best estimate of true DBP?

The disappearance of Korotkoff sounds following initial muffling

Which area of the fundus is the central focal point for incoming images?

The fovea The fovea is the area of the retina which is responsible for central vision. It is surrounded by the macula, which is responsible for more peripheral vision. The optic disc and physiologic cup are where the optic nerve enters the eye.

152. During a sports physical, you note that the vision of an 18-year-old male athlete is 20/30 in both eyes. Which of the following statements is true?

The patient can see at 20ft what a person with normal vision can see at 30ft.

313. What is the clinical significance of this finding in a 35-year-old patient?

The patient should be evaluated for hyperlipidemia Arcus senilis is caused by lipid deposits deep in the edge of the cornea and is quite commonly present in the elderly. However, it can also appear earlier in life as a result of hypercholesterolemia.

337. Which of the following is the correct statement regarding the size of the arterioles and veins on the fundus of the eye?

The veins are larger than the arterioles The fundus of the eye is opposite the lens, and includes the retina, optic disc, macula and fovea, and posterior pole. The eye's fundus is the only part of the human body where the microcirculation can be observed directly. The retinal arteries and veins emerge from the nasal side (left) of the optic disc. Vessels directed temporally have an arching course; those directed nasally have a radial course. Arteries are brighter red and narrower than veins.

68yo retired college professor presents for routine physical exam, after pt has been reading a novel in waiting room for ~20 mins, tech records BP in both arms using automated device, tech notes 20mmHg different in SBP between right & left arms, reports readings 10 mins later & records the same asymmetrical SBP. Which of the following is true regarding this physical finding?

This finding is clearly abnormal and requires immediate evaluation for possible cardiovascular emergency BP difference of 10-15mmHg between right and left arm should be recognized as abnormal

285. During a sports physical exam, a 16-year-old patient is noted to have a few beats of horizontal nystagmus on extreme lateral gaze that disappear when the eyes move back toward midline. Which statement best describes this clinical finding?

This is a normal finding Horizontal nystagmus is a normal variation on physical exam. Full movement of the eyes is controlled by the integrated function of cranial nerves III (oculomotor), IV (trochlear), and VI (abducens). Holding the patient's chin to prevent movement of the head, ask the patient to watch your finger as it moves through the six cardinal fields of gaze. Then ask the patient to look to the extreme lateral (temporal) positions. A few horizontal nystagmic beats are within normal limits (WNL).

87F generally healthy and cognitively sharp complains to clinician of slow loss of vision, with similar problems in both eyes, particularly when she looks straight ahead. she is having difficulty reading of late. what is a reasonable response to her?

This may be the onset of macular degeneration which can ophthalmologist should confirm results in central vision loss

The components of health history include of the following except which?

Thorax and lungs This is part of physical examination

Susanne is a 27 year old who has had headaches, muscle aches, and fatigue for the last 2 months. You have completed a thorough history, examination, and laboratory workup but have not found a cause. What would your next action be?

To screen for depression Although you may consider referrals to help with the diagnosis and treatment for this patient, screening is a time-efficient way to recognize depression. This will allow her to be treated more expediently. You may tell the patient you have not found an answer yet, but you must also tell her that you will not stop looking until you have helped her.

58. The best test for diagnosing glaucoma is which of the following?

Tonometry A tonometer is used to measure the intraocular pressure (IOP) of the eye to screen for glaucoma. Normal range IOP is 10 to 22 mmHg.

A young man is concerned about a hard mass he has just noticed in the midline of his palate. On examination, it is indeed hard and in the midline. There are no mucosal abnormalities associated with this lesion. He is experiencing no other symptoms. What will you tell him is the most likely diagnosis?

Torus palatinus Torus palatinus is relatively common and benign but can go unnoticed by the patient for many years. The appearance of a bony mass can be concerning. Leukoplakia is a white lesion on the mucosal surfaces corresponding to chronic mechanical or chemical irritation. It can be premalignant. Thrush is usually painful and is seen in immunosuppressed patients or those taking inhaled steroids for COPD or asthma. Kaposi's sarcoma is usually seen in HIV-positive individuals and is classically a deep purple.

Which of the following percussion notes would you obtain over the gastric bubble? A) Resonance B) Tympany C) Hyperresonance D) Flatness

Tympany The gastric bubble produces one of the longest percussion notes. A patient with COPD may have hyperresonance over his chest, while a normal person would have resonance. Dullness is heard over a normal liver, and flatness is heard if one percusses a large muscle.

710. A 30-year-old woman with mild persistent asthma and allergic rhinitis is seen in an urgent care clinic for complaints of shortness of breath and wheezing. She reports using her albuterol inhaler two to three times per day for 4 days the previous week. She reports waking up for 3 nights due to wheezing the past week. She had previously been prescribed low-dose flunisolide (AeroBid), two inhalations twice per day. Vital signs reveal a temperature of 99.0°F, pulse of 88 beats/min, and respiratory rate of 14 breaths/min. Which of the following actions is the next step?

Use the spirometer to assess severity of symptoms The patient is having an acute asthmatic exacerbation. The patient's vital signs reveal a respiratory rate of 14 breaths/min and pulse of 88 beats/min, which are within normal limits (not in respiratory distress). Before administering nebulized albuterol, use the spirometer to quantify the degree of impairment and listen to the breath sounds. Then after the treatment, recheck spirometry to evaluate the patient's response. The nebulizer treatment can be repeated in 20 minutes as needed. Change the patient's prescription to a combination steroid and long-acting beta-agonist inhaler plus a Medrol dose pack.

17M presents to STD clinic at behest of his brother, convinced pt to go after he disclosed he prefers homosexual partners but is afraid his last partner may have given him an infection, expresses to intake nurse that he is unashamed of sexual orientation & will not stay through visit if he feels he is dismissed or discriminated against because of it. NP receives this communication prior to entering the exam room & decides to employ active listening to best connect with pt at critical juncture in his care with the clinic, which is an active listening technique?

Using nonverbal communication to encourage pt to expand their narrative Demands such skills as setting aside diagnostic priorities in favor of open discussion; using verbal & non-verbal skills to encourage pt to engage fully with their own narrative, being aware of pts emotional state

29F professional athlete presents co chronic menstrual complaints, remarks to nursing staff in the past she has experienced a dismissal of her complaints bc of her high level of physical fitness & conditioning. She is seeking a care provider who will explore the issue in more detail & work with her particular concerns. Which of the following is the description of pt-centered care this individual seeks?

Validating and empathetic with open-ended questions Patient-centered care is loosely structured, supportive, aimed at meeting pt where they are in their disease process, characterized by open-ended questions

A patient tells you about her experience with prolonged therapy for her breast cancer. You comment, "That must have been a very trying time for you." What is this an example of?

Validation

246. What type of breath sounds are best heard over the base of the lungs?

Vesicular breath sounds Vesicular breath sounds are heard best over the base of the lungs. Vesicular sounds are soft and/or blowing, heard throughout inspiration, and fade away with expiration. Bronchial sounds are heard over the bronchi, the largest tubes in the anterior chest. Sounds are loud and high pitched. Tracheal breath sounds are heard over the trachea. These sound harsh and similar to air being blown through a pipe.

475. Auscultation of normal breath sounds of the chest will reveal:

Vesicular breath sounds in the lower lobe Normal sounds of the chest wall include vesicular breath sounds in the lower lobes. Bronchial breath sounds are heard best at the second and third intercostal spaces. Tracheal breath sounds are heard over the trachea.

41. A mother brings her 4-year-old daughter, who just started attending preschool, to the health clinic. She tells the nurse practitioner that her child is complaining of burning and itching that started in the left eye. Within 2 days it involved both eyes, and the child developed a runny nose and sore throat. During the physical exam, the child's eyes appear injected bilaterally with no purulent discharge. The throat is red, the inferior nasal turbinates are swollen, and lymph nodes are palpable in front of each ear. Which of the following is most likely?

Viral conjunctivitis Viral conjunctivitis The causative organisms of viral conjunctivitis (pink eye) include adenovirus and other virus types. It can present with or without cold symptoms. Patient complains of itchy red eyes and may have clear discharge accompanied by preauricular lymphadenopathy. Does not affect vision. If a contact lens wearer, assume bacterial infection and obtain culture of eye discharge.

160. All of the following are factors important in determining the peak expiratory flow volume except:

Weight

428. Peak expiratory flow (PEF) meters are used to monitor asthma by using personal best measurements. All of the following factors are used to determine the PEF except:

Weight

A 75-year-old homemaker brings her 76-year-old husband to your clinic. She states that 4 months ago he had a stroke and ever since she has been frustrated with his problems with communication. They were at a restaurant after church one Sunday when he suddenly became quiet. When she realized something was wrong he was taken to the hospital by EMS. He spent 2 weeks in the hospital with right-sided weakness and difficulty speaking. After hospitalization he was in a rehab center, where he regained the ability to walk and most of the use of his right hand. He also began to speak more, but she says that much of the time "he doesn't make any sense." She gives an example that when she reminded him the car needed to be serviced he told her "I will change the Kool-Aid out of the sink myself with the ludrip." She says that these sayings are becoming frustrating. She wants you to tell her what is wrong and what you can do about it. While you write up a consult to neurology, you describe the syndrome to her. What type of aphasia does he have?

Wernicke's aphasia

Mr. Q. is a 45-year-old salesman who comes to your office for evaluation of fatigue. He has come to the office many times in the past with a variety of injuries, and you suspect that he has a problem with alcohol. Which one of the following questions will be most helpful in diagnosing this problem?

When was your last drink?

pt with CF has been co fullness in his left nasal cavity, exam of his nose using an otoscope and speculum reveals a normal nasal septum bu pale, saclike growth of inflamed tissue that is obstructing a large part of the nasal cavity, what is the most likely dx?

nasal polyp more likely in CF

683. An obese middle-aged man with type 2 diabetes is diagnosed with familial hypercholesterolemia. During the physical exam, the nurse practitioner notices that the patient has raised, sharply demarcated yellowish patches on the inner canthi of both eyes and on the upper and lower eyelids that are symmetrical and nontender. Which of the following conditions is being described?

Xanthelasma

Despite having high BP readings in the office, Mr. Kelly tells you that his readings at home are much lower. He checks them twice a day at the same time of day and has kept a log. How do you respond?

You question the accuracy of his measurements

65yo overweight male presents at the clinic with hoarseness which has lasted for around 2 months, he thinks it began along with a cold. he is not feeling badly other than frequent heartburn, and he has continued to work as a bartender (for the past 30 years), but he is having difficulty being heard and understood because of his hoarse voice. a diagnosis that is on the differential list includes which of the following?

acid reflux can cause hoarseness through damage to laryngeal area due to acid stomach contents being refluxed to the vocal cords

the following information is best placed in which category? "the patient was treated for an asthma exacerbation in the hospital last year, the pt has never been intubated"

adult illnesses

the following information is best placed in which category? "the pt had stent placed in left anterior descending artery in 1999"

adult illnesses

Alexandra is a 28-year-old editor who presents to the clinic with abdominal pain. The pain is a dull ache, located in the right upper quadrant, that she rates as a 3 at the least and an 8 at the worst. The pain started a few weeks ago, it lasts for 2 to 3 hours at a time, it comes and goes, and it seems to be worse a couple of hours after eating. She has noticed that it starts after eating greasy foods, so she has cut down on these as much as she can. Initially it occurred once a week, but now it is occurring every other day. Nothing makes it better. From this description, which of the seven attributes of a symptom has been omitted?

associated manifestations

82M seems to speaking loudly during examination, suggesting that he may not be hearing well, what is a good question to ask him to help identify whether or not he has hearing loss?

how well does he understand people in noisy environments such as a restaurant?

One important examination technique involves using 3rd fingers of each hand to determine health of internal organs, it is called

percussion

35yo female patient has had migraines for much of her adult life. At her regular checkup, she is healthy, takes no medications, except oral contraceptives, exercises and has a steady job. Her only complaint is that her migraines seem to have become worse, and, for the past few weeks, she has been waking up at night with headache and also nausea. Which of the following is the best course of action?

take a further history and perform a very careful neurological examination concerned for increased ICP from tumor or other mass

A 55yo woman with a headache explains to the clinician that she has had headaches before, but this one is unusual because of some new symptoms. Which of the following symptoms would prompt an immediate investigation?

the patient also developed a fever and night sweats and thinks she lost some weight

25yo construction worker co swishing noise in both ears that never goes away and has occurred for about 6mos. he is otherwise healthy, is able to work on his job (operating large, vibrating machinery) without problems, and is not taking any meds. A complete exam reveals an abnormality, which of the following is most often associated with tinnitus?

vertigo and tinnitus together comprise a syndrome called Meniere Disease, which may affect young adults


Related study sets

ExamFX - Completing the Application, Underwriting, and Delivering the Policy

View Set

Chp 7: High-Risk Antepartum Nursing Care

View Set

Fundamentals: josphine, kim, mona

View Set

Vertical Asymptotes of Rational Functions

View Set

U.S. History & Geography: Independence for Texas, Chapter 7 Lesson 3

View Set

AC305 Chapter 3 Practice Questions

View Set

Cancer and Enzymes Assignment Assignment

View Set

Chapter 9 - Production and Operations Management

View Set